Aiims Adr Plexus Ent

March 18, 2018 | Author: kishorechandra | Category: Ear, Larynx, Hearing Loss, Hearing, Human Head And Neck


Comments



Description

1.A middle aged lady with Rt sided thyroid gland underwent hemithyroidectomy of the right side. After the surgery, she had weak voice. Laryngoscopy was done. There is loss of tension of the right vocal cord and the right cord was flapping with respiration .Laryngoscopy revealed the below finding.Which of the following probably happened- A. Rt recurrent laryngeal N. was damaged B. Rt superior laryngeal N. was damaged C. Both the nerves were damaged D. This is a normal finding after hermithyroidectomy 2. An adult female patient has a history of foul smelling ear discharge from the right side for the past 8 months. She now has fever for the last 3 days. The fever is irregular associated with chills and rigor, patient is also complaining of headache. Which of the following is the most probable complication- A. Labryrinthine fistula B. Lateral sinus thrombophlebitis C. Labyrinthitis D. Temporal lobe abscess 3. A young man met with an accident leading to loss of hearing in the right ear. On otoscopy examination the tympanic membrane was intact .Pure tone audiometry showed AB gap of 55 dB on the right ear with normal cochlear reserve, which of the following will be probable tympanometry finding- A. As type tympanogram B. Ad type tympanogram C. B type tympanogram D. C type tympanogram 4. Type V curve of Bekesy audiometry is seen in- A. Retrocochlear lesion B. Cochlear lesion C. Malingerers D. Normal hearers 5. A 35 year old man with unsafe CSOM(cholesteatoma)was operated one year back.The surgery done was MRM (Intact canal wall mastoidectomy).He has recurrence of the disease (cholesteatoma what would be the most imp. Step): A. Antibiotics and close follow up B. Radical mastoidectomy C. Canal wall down MRM D. Cortical mastoidectomy 6. A 3 year old child presents with fever, sore throat and has difficulty in breathing. His radiograph is given below. What is the most likely diagnosis? A. TB larynx B. Croup C. Acute Epiglottitis D. Laryngomalacia 7. A 6 year old child complains of hearing loss in both ears with sleeping with open mouth. He had retracted and bluish TM in both of the ears. You have planned to do surgery. Which of the following would you not do pre-operatively- A. Culture and sensitivity of fluid aspirated from middle ear B. Impedance audiometry C. X-ray of naso-pharynx->adenoid view D. All should be done 8. Identify the instrument: A. Luc’s forceps B. Mollison’s mastoid retractor C. Eve’s Tonsil Snare D. Walsham forceps 9. In Auditory brainstem response tracing wave III corresponds to A. Cochlear nucleus B. Superior olivary complex C. Lateral lemniscus D. Inferior colliculus 10. Unilateral pain in the ear in the absence of ear disease can be due to all except: A. Allergic rhinitis B. Peritonsillar abscess C. Cancer of the pyriform fossa D. Temporomandibular joint dysfunction 11. A young boy of 18 years gets recurrent busts of otitis externa after swimming. Which of the following methods should be recommended? A. Not to swim B. Use 2% acetic acid after swimming C. Use ear drops containing an antibiotic and steroid D. Use an antifungal ear drops after swimming 12. Which of the following conditions predispose to otitis media with effusion? A. Cleft palate B. Down's syndrome C. Carcinoma nasopharynx D. All the above 13. A 35 yr. male presents with the only c/o hoarseness of voice for the past 2 yrs. He has been a chronic smoker for 30 yrs, o/e a reddish area of mucosal irregularity overlying a portion of both cords are seen. Management would include all except- A. Cessation of smoking B. Bilateral cordectomy C. Microlaryngeal surgery for biopsy D. Regular follow up 14. A 55 year old male presents will sudden SN hearing loss (Rt) side with tinnitus on the same side.Which of the following is the most imp. Investigation- A. Gadolium MRI B. CT scan C. Ear examination under microscopy D. None of the above 15. Which of the following is the commonest presentation of septal hematoma- A. Unilateral nasal obstruction B. Bilateral obstruction C. Unilateral nasal bleeding D. Bilateral nasal bleeding 16. In superior orbital Fissure syndrome all of the following cranial nerves may be involved except- A. III B. IV C. V D. VI 17. Which of these may be indirect laryngoscopic findings in the patient of phonasthenia- A. Elliptical space between vocal cords (Thyroaretenoid involved) B. Triangular space in posterior glottis (Inter-aretenoid involved) C. Key hole appearance of glottis (both muscle involved) D. All of the above 18. The facial nerve is damaged at the 2nd genu in the middle ear leading to facial nerve palsy on the right side. Which of these will be the stapedial reflex finding when sound stimulus is given on the right side- A. Reflex will be absent on the ipsilateral side only B. Reflex will be absent on the contralateral side only C. Reflex will be absent on both the sides D. Reflex will be present on both the sides 19. A 7-year-old child developed acute otitis media. He was treated with antibiotics for 10 days. His pain and fever subsided completely but still had conductive hearing loss. Your next line of treatment is: A. Give another course of a different antibiotic B. Do a myringotomy and culture the middle ear fluid C. Do a myringotomy and insert a grommet D. Wait and watch for 3 months for fluid to drain spontaneously 20. MC Governs Technique may be a life saving technique during which of the following- A. Difficult tracheostomy B. Difficult intubation C. B/L Choanal atresia D. CSF rhinorrhea 21. During expiration, maximum amount of air is passed through which of these meatus- A. Interior meatus B. Middle meatus C. Superior meatus D. Supreme meatus 22. Long-standing case of otitis media with effusion can develop all of the following complications except: A. Retraction pockets B. Cholesteatoma C. Ossicular fixation D. Cholesterol granuloma 23. A 5-year-oId male child had .acute otitis being treated with ear drops, oral antibiotics and analgesics. Two weeks after he presented with a swelling over the mastoid, pain in the ear with pulsatile ear discharge and fever. Now treatment of this child would include all except: A. W antibiotics B. Cortical mastoidectomy C. Modified radical mastoidectomy D. Analgesics 24. Which of these above at paranasal sinus in true- A. In children ethmoidal sinusitis is more common B. Polyps are most common in maxillary sines C. PNS malignancy is more common in Ethmoidal sinus D. Mucocele is more common in splenoid sinus 25. Which of these secretary glands is not supplied by fibres from facial nerve- A. Parotid gland B. Lacrimal gland C. Submandibular gland D. Sublingual gland 26. A pregnant woman in third trimester complains of hearing her own sounds. Examination of the ear shows movements of tympanic membrane synchronous with respiration and especially exaggerated when nostril on the contralateral side is occluded. Your diagnosis is: A. Eustachian tube obstruction B. Patulous eustachian tube C. Otitis media with effusion D. Otosclerosis 27. Which of these nerves is closely related to sphenoid sinus(Anatomically)- A. Ophthalmic nerve B. Optic nerve C. Oculomotor nerve D. Trochlear nerve 28. Regarding glomus tumour of middle ear which of the following statements is not correct? A. More common in men B. Grows very slowly C. Diagnostic biopsy is contraindicated D. Multicentric origin 29. Bulge on the roof of ear is called- A. Promontory B. Arcuate eminence C. Tegmen Tympan D. None 30. A 45 year old person had Rt enlarged tonsil for 6 weeks. Tonsillectomy was done for this what should be the next logical step- A. Sent the specimen for histo-pathological Examination B. Sent the specimen for culture sensitivity examination C. Take a swab from tonsillar bed for culture sensitivity examination D. Styloid process should be removed 31. Which of these about nasal packing is true- A. After nasal packing, patient always be deeply sedated B. Only the bleeding nasal cavity should be packed C. Both anterior and posterior nasal packing may be done together on the patient D. Nasal pack should be changed after every 24 hrs 32. Which of the following signs is not 8asoclated with glomus tumour? A. Rising sun appearance B. Aquino's sign C. Brown's sign D. Griesinger's sign 33. Contributing arches for the development of pinna- A. First arch B. First arch and second arch C. Second and third arch D. Fourth arch 34. External auditory canal is supplied by all except- A. Trigeminal B. Vagus C. Facial D. Glossopharyngeal 35. The external landmark for mastoid antrum is- A. Concha B. Scaphoid fossa C. Cymba concha D. Triangular fossa 36. Acute masoiditis will cause the following- A. Tragnal sign positive B. Tenderness at Cymba conchae C. Both D. None 37. Tympanic membrane attains it angulation of 55* at- A. Birth B. 4-5 years C. 5-6 years D. 7 years 38. Treatment of choice for a glomus tumour restricted to promontory of middle ear is: A. Surgical removal B. Embolisation C. Radiation therapy D. Wait and watch 39. Thickness of lateral wall of mastoid antrum at birth- A. 1 mm B. 2 mm C. 4 mm D. 5 mm 40. Eustachian tube opens- A. Above the posterior end of the inferior turbinate B. Below the inferior turbinate C. Below and behind the inferior terbinate D. Behind the middle turbinate 41. Apex of the cocheal perceives- A. Low frequency sounds B. High frequency sounds C. Both D. None 42. Which of the following is not the site of paraganglioma? A. Carotid bifurcation B. Jugular foramen C. Promontory of middle ear D. Geniculate ganglion 43. All are approaches to petrous apex except- A. Ramadier’s B. Rosen’s C. Almoor’s D. Frenckner’s 44. Commonest site for cholesteatoma- A. Posterior mesotympanum B. Prussak’s space C. Anterior epitympanum D. Protympanum 45. Positive fistula sign is seen- A. Otosclerosis B. Meniere’s disease C. C.S.O.M D. Dead labyrinth 46. Brown’s sign seen is- A. Acoustic neuroma B. Glomusjugulare C. Congenital cholestroma D. Meningioma 47. A 38 year old man had labyrinthine fistula with positive fistula sign.Two weeks later fistula sign became negative without any treatment.Which of the following will be the rinne’s test(tunning fork) finding- A. True positive rinne’s test B. False positive rinne’s test C. True negative rinne’s test D. False negative rinne’s test 48. A male presents with dull TM, deafness and tinnitus, type B curve impedence and neck mass, the most likely cause is- A. Canasopharynx B. Ca middle ear C. TB of neck nodes D. Cholesteatoma 49. CROS hearing aids is used for- A. Bilateral severe deafness B. Severe unilateral deafness C. Mild neural deafness D. None of the above 50. Which of the following is true about the complications of acute mastoiditis except- A. The most common abscess is post/retro aural abscess B. Luc’s abscess is in the superior canal wall C. Bezold’s abscess is usually subperiosteal D. Zygomatic abscess is usually subperiosteal 51. Luc’s abscess is seen in- A. Behind the mastoid B. In the external auditory canal C. In the sheath of sternocleidomastoid D. None of the above 52. Treatment of stag IV carcinoma larynx is- A. Surgery B. Chemotherapy C. Surgery + Radiotherapy D. Surgery Chemotherapy 53. Early vocal nodules are best treated by- A. Microlaryngeal surgery B. Laser excision C. Voice rest D. Speech therapy 54. Most common fungal infection of the paranasal sinus- A. Rhinosporidiosis B. Mucormycosis C. Aspergillosis D. Monilia 55. Aim of mastoid surgery In CSOM which should receive first priority is: A. Making the ear dry B. Improvement in hearing C. Preservation of hearing D. Rendering the ear safe 56. Hypoactive labyrinth cause nystagmus to- A. Same side B. Opposite side C. Rotatory nystagmus D. No-nystagmus 57. Vestibular schwannoma 1 cm in diameter, the investigation of choice is- A. CT scan B. MRI C. Angiography D. Plain X-ray 58. A 30-year-old woman with family history of hearing loss from her mother's side developed hearing problem during pregnancy. Hearing loss is bilateral slowly progressive, with bilateral tinnitus that bothers her at night. Pure tone audiometry shows conductive hearing loss with an apparent bone conduction hearing loss at 2000 Hz. What is the most likely diagnosis? A. Otosclerosis B. Acoustic neuroma C. Otitis media with effusion D. Sigmoid sinus thrombosis 59. The prevalence of clinical otosclerosis is highest In: A. Japanese B. Caucasians C. Americans D. Africans 60. Accessory ostia in the lateral wall are located in the- A. Middle meatus B. Posterior fontanelle C. Inferior meatus D. None of the above 61. 45 year old diabetic patient presents with the history of black necrotic feling up the nasal cavity and diplopia of the affected side,the treatment of choice is- A. FESS B. Endoscopic clearance C. Local debridement D. Local debridement with orbital exenteration 62. Which of the following tracheostomy tubes have double cuffs- A. Shiley’s tube B. Salpekar tube C. Montgomery tube D. Keonig’s tube 63. Otosclerosis is: A. Autosomal dominant B. Autosomal recessive C. X-linked disease D. Mitochondrial disorder 64. Extension of angiofibroma to which of these areas may require pre-operative radiotherapy- A. Intra cranial extension B. Nasal extension C. Pterygopalaine fossa extension D. None of the above 65. Which of the following is not a part of temporal bone- A. Squamous bone B. Occipital bone C. Petrous bone D. Tympanic plate 66. Most common site of otosclerosis is- A. Oval window B. Round window C. Fissula ante fenestram D. None 67. Most common site for Initiation of stapedial otosclerosis is: A. Fissula ante-fenestram B. Fossula post-fenestram C. Footplate of stapes D. Margins of stapes 68. Endoscopic sinus surgery is used for which of the following- A. Antrochoanal polyp removal B. Repair of CSF rhinorrhea C. Optic nerve decompression D. All 69. Drug of choice for allergic fungal sinusitis is- A. I/U amphotericin B B. Antibiotics C. Corticosteroids D. None 70. A 10-yr old boy is having sensory neural deafness. He showed no improvement with conventional hearing aids. Most appropriate management is- A. Bone conduction hearing aids B. Fenestration C. Stapes fixation D. Cochlear implant 71. The following is difficult to see during indirect laryngoscopy- A. True cords B. False cord C. Posterior commissure D. Anterior commissure 72. Pneumatized middle turbinate is called- A. Bulla ethmoidalis B. Concha bullosa C. Onodi cell D. Heller cell 73. Genetic defect leading to complete failure of inner ear development is of the following type- A. Michel aplasia B. Mondini aplasia C. Scheibe aplasia D. Alexander aplasia 74. In the pathogenesis of otosclerosis, the disease process starts in: A. Periosteal layer of otic capsule B. Endosteal layer of otic capsule C. "Bone of otic capsule which develops from cartilage D. Mucoperiosteum of the promontory 75. Which of the following statements Is not true about stapedial otosclerosis? A. Slowly progressive conductive hearing loss with normal tympanic membrane B. Rinne's test is negative C. Tympanogram is Ao type D. Eustachian tube is patent 76. Which of the following type of tympanoplasty is called columellatympanoplasty- A. Type I B. Type II C. Type III D. type IV 77. Which of the following is the newer device in the treatment of meniere’s disease- A. P-100 B. mienette’s device C. Both D. None 78. Schwartze's sign is: A. Swelling over the mastoid B. Reddish hue seen in the hypotympanum behind an intact tympanic membrane C. Improved hearing in noisy surroundings D. Reddish hue seen over the promontory 79. A 45 years old lady presented with a swelling in the parotid gland for the past 6 months .FNAC done from this area revealed lympho-epithelioma.All are true of lympho-epithelioma except- A. Parotid gland is the most common site of lympho- epithelioma in the head and neck region B. It is known to be caused by EBV C. The epithelial cells are mainly malignant with lymphoid tissue D. It is a highly radio sensitive tumour 80. Singular nerve supplies which of the following semicircular canals- A. Superior canal B. Posterior canal C. Lateral canal D. There is no such nerve in the inner ear 81. A 30 year old lady with conductive hearing loss on atoscopy tympanic membrane has flamingo pink blush. Impedance audiometry shows AS curve the treatment of choice is- A. NaF B. Stapedectomy C. Fenestration surgery D. Wait and watch 82. Which of the following is seen in posterior rhinoscopy- A. Vomer B. Inferior turbinate C. Eustachian tube opening D. All 83. Holman miller sign is seen in- A. Angiofibroma B. Nasopharyngeal Ca C. Antrochoneal polyp D. All 84. In an otosclerotic patient Rinne's test was negative with a tuning fork of 512 Hz but positive for 1024 Hz. His minimum predicted AB gap on audiometry would be: A. 20dB B. 30dB C. 45 dB D. 60 dB 85. The triad constituting syndrome of Van der Hoeve Includes all except: A. Osteogenesis imperfecta B. Conductive hearing loss C. Blue sclera D. Preauricular sinuses 86. Carhart's notch In audiogram Is deepest at the frequency of: A. 0.5 kHz B. 2.0 kHz C. 4.0 kHz D. 8.0 kHz 87. All of the following statements are true about Carhart's notch except: A. It is a sensorineural hearing loss B. Maximum loss is centred at 2 kHz C. Seen only in stapes fixation D. Cannot by reversed 88. Treatment of choice for otosclerosis In a young person employed In office is: A. Hearing aid B. Stapedectomy C. Stapes mobilisation only D. Fenestration operation 89. Which ossicle is most commonly damaged by cholesteatoma- A. Malleus B. Incus C. Stapes D. All have equal percentage 90. Condition in which loud sounds produce giddiness is called: A. Paracusis Willisi B. Hennebert's sign C. Tullio phenomenon D. Otolithic crisis of Tumarkin 91. A 31-year-old female patient complains of bilateral Impairment of hearing for the past 5 years. On examination, tympanic membrane (TM) is normal and audiogram shows a bilateral conductive loss. Impedance audiometry shows AS type of curve and absent acoustic reflex. All constitute part of treatment except: A. Hearing aid B. Stapedectomy C. Sodium fluoride D. Gentamicin therapy 92. During otoscopy of patients right ear otoscopy should ideally be held in which hand- A. Right hand B. Left hand C. Either D. None 93. Which of the following lesions is pre-cancerous- A. Keratosis of tonsil B. Keratosis of larynx C. Aphthous ulcers D. All 94. A 31-year-old female patient complains of bilateral Impairment of hearing for the past 5 years. On examination, tympanic membrane (TM) is normal and audiogram shows a bilateral conductive loss. Impedance audiometry shows AS type of curve and absent acoustic reflex. All constitute part of treatment except: A. Hearing aid B. Stapedectomy C. Sodium fluoride D. Gentamicin therapy 95. Most common cause for bilateral conductive deafness in a child is: A. Otosclerosis B. Otitis media with effusion (OME) C. Acute otitis media D. Congenital cholesteatoma 96. Ramavati is 40 years old female presented with a progressively increasing lump in the parotid region. On oral examinations, the tonsil was pushed medially. Biopsy showed it to be pleomorphic adenoma. The appropriate treatment is- A. Superficial parotidectomy B. Lumpectomy C. Conservative total parotidectomy D. Enucleation 97. Which of the following is not an indication of radiotherapy in Pleomorphic adenoma of parotid- A. Involvement of deep lobe B. 2nd histologically benign recurrence C. Microscopically positive margins D. Malignant transformation 98. Nerve which lies in association to Warton’s duct is- A. Hypoglossal B. Lingual C. Lingual nerve D. Mandibular branch of facial nerve 99. A 55-year-old female presents with tinnitus, dizziness and history of progressive deafness. Differential diagnosis Includes all except: A. Acoustic neuroma B. Endolymphatic hydrops C. Meningioma D. Histiocytosis X 100. Bilateral destruction of auditory cortex leads to- A. Total deafness B. Hearing defect for higher frequency sounds C. Deficiency in interpretation of sounds D. Inability for orientation of sounds 101. Tympanic membrane develops from: A. Ectoderm B. Mesoderm C. Endoderm D. All the three germinal layers 102. Number of centres from which bony labyrinth ossifies are: A. 6 B. 8 C. 10 D. 14 103. Nerve supply of tympanic membrane is derived from all except A. Auriculotemporal nerve B. Auricular branch of vagus C. Great auricular nerve D. Glossopharyngeal nerve 104. Area of stapes footplate is: A. 1.5 sq mm B. 2.2 sq mm C. 3.0 sq mm D. 3.2 sq mm 105. The pinna attains 90-95% of adult size by: A. Birth B. 5-6 years C. 9-10 years D. 11-12 years 106. Sensory nerve supply of the middle ear comes from: A. Vagus nerve B. Caroticotympanic nerves C. Glossopharyngeal nerves D. Chorda tympani 107. Citelli's angle is: A. Solid angle B. Cerebellopontine angle C. Sinodural angle D. Genu of facial nerve 108. In a normal ear, which of the following is a false statement? A. Total length of external ear canal is 36 mm B. Pinna has to be pulled upwards and backwards to see the tympanic membrane C. External ear canal does not contain ceruminous glands or hair follicles in the deep bony part D. Dehiscences may be seen in outer cartilaginous canal 109. Which of the following statements is true? A. Korner's septum in the mastoid separates squamous cells from the deeper petrosal cells B. Facial recess lies medial to the sinus tympani C. Trautmann's triangle forms an important surgical landmark to locate endolymphatic sac D. Arcuate eminence is landmark for lateral semicircular canal 110. Which of the following is not true about the appearance of tympanic membrane? A. A red tympanic membrane may be normal in a crying child B. A retracted tympanic membrane shows prominent lateral process of malleus and foreshortened handle of malleus C. A bulging tympanic membrane loses all landmarks D. Creation of positive or negative pressure with Siegel's speculum or pneumatic otoscope has no effect onmovement of normal tympanic membrane 111. During superficial parotidectomy, the most reliable landmark to Identify main trunk of facial nerve is: A. Mastoid tip B. Styloid process C. Tympanomastoid suture D. Cartilage of external auditory canal 112. Endolymph is formed in: A. Utricle B. Endolymphatic sac C. Scala media D. Scala tympani 113. Communication between middle ear and eustachian tube is obliterated surgically in: A. Cortical mastoidectomy B. Modified radical mastoidectomy C. Radical mastoidectomy D. Bondy's mastoidectomy 114. Stapes superstructure develops from: A. Meckel's cartilage B. Reichert's cartilage C. Both (a) and (b) D. Both (a) and (b) plus bony otic capsule 115. Interpretation of SERA (brain-stem evoked response audiometry) is affected by: A. Age of the child B. Sex of the child C. Sedation D. Sleep 116. Inner ear malformation in fetus can occur when mother during pregnancy is exposed to all except: A. Radiation B. German measles C. Cytomegalovirus D. Lithium 117. Cause of sensorineural hearing loss due to furosemide toxicity is: A. Damage to outer cells of cochlea B. Damage to inner cells of cochlea C. Stria vascularis D. Cochlear nerve 118. All are true about Jacobson's nerve except: A. It is a branch of superior ganglion of vagus B. Supplies middle ear and mastoid air cells C. Supplies secretomotor fibres to parotid D. Section of this nerve relieves gustatory sweating 119. In a sitting position with head tilted 60 0 backward, which of the following canals is stimulated during caloric testing: A. Superior B. Posterior C. Lateral D. Both lateral and superior 120. Dorello's canal transmits which of the following nerves? A. Ophthalmic division of CN V B. Abducens (CN VI) C. Facial nerve (CN VII) D. Statoacoustic nerve (CN VIII) 121. Costen's syndrome is characterised by all except: A. Otalgia B. Vertigo C. Tinnitus D. Recurrent dislocation of temporomandibular joint 122. Operation of choice for coalescent mastoiditis is: A. Cortical mastoidectomy B. Modified radical mastoidectomy C. Radical mastoidectomy D. Fenestration operation 123. The cough response caused while cleaning the ear canal is mediated by stimulation of: A. The V cranial nerve B. Innervation of external ear canal by C1 and C2 C. The X cranial nerve D. Branches of the VII cranial nerve 124. A 38-year-old gentleman reports of decreased hearing in the right ear for the last 2 years. On testing with a 512-Hz tuning fork, the Rinne's test (without masking) is negative on the right ear and positive on the left ear. With the Weber's test the tone is perceived louder in the left ear. The patient most likely has: A. Right conductive hearing loss B. Right sensorineural hearing loss C. Left sensorineural hearing loss D. Left conductive hearing loss 125. Which of the following Is not a typical feature of Meniere's disease? A. Sensorineural deafness B. Pulsatile tinnitus C. Vertigo D. Fluctuating deafness 126. Which of the following is not a typical feature of malignant otitis externa? A. Caused by Pseudomonas aeruginosa B. Patients are usually old C. Mitotic figures are high D. Patient is immune compromised 127. Treatment of choice for glue ear is: A. Conservative B. Myringotomy with cold knife C. Myringotomy with ventilation tube insertion D. Myringotomy with diode laser 128. Which of Ute following is true regarding facial nerve palsy associated with temporal bone fracture? A. Common with longitudinal fracture B. Common with transverse fracture C. Always associated with CSF otorrhoea D. Facial nerve injury is always complete 129. The posterosuperior retraction pocket, if allowed to progress, will lead to: A. Sensorineural hearing loss B. Secondary cholesteatoma C. Tympanosclerosis D. Tertiary cholesteatoma 130. A 30-year-old male is having attic cholesteatoma of left ear with lateral sinus thrombophlebitis. Which of the following will be the operation of choice? A. An intact canal wall mastoidectomy B. Simple mastoidectomy with tympanoplasty C. Canal wall down mastoidectomy D. Mastoidectomy with cavity obliteration 131. Which is the Investigation of choice in assessing hearing loss in neonates? A. Impedance audiometry B. Brain-stem evoked response audiometry C. Free field audiometry D. Behavioural audiometry 132. Which of the following conditions causes the maximum hearing loss? A. Ossicular disruption with intact tympanic membrane B. Disruption of malleus and incus as well tympanic membrane C. Partial fixation of the stapes footplate D. Otitis media with effusion 133. Use of Siegel's speculum during examination of the ear provides all except: A. Magnification B. Assessment of movement of the tympanic membrane C. Removal of foreign body from the ear D. As applicator for the powdered antibiotic to ear 134. All are true for Gradenigo's syndrome except: A. It is associated with conductive hearing loss B. It is caused by an abscess in the petrous apex C. It leads to involvement of the cranial nerves V and VI D. It is characterized by retro-orbital pain 135. In right middle ear pathology, Weber's test will be: A. Normal B. Centralized C. Lateralised to right side D. Lateralised to left side 136. A 25-year-old woman suffering from bilateral hearing loss for 6 years which became profound with pregnancy. On tympanogram, which of the following curve is obtained? A. AD B. As C. B D. A 137. Unit of Intensity while testing for threshold of hearing in an audiogram is: A. dB SL B. dB HL C. dB A D. dB SPL E. dBnHL 138. Otoacoustic emissions are produced by: A. Inner hair cells B. Outer hair cells C. Basilar membrane D. Auditory nerve 139. Speech frequencies Include: A. 125 250 500 Hz B. 250 500 1000 Hz C. 500 1000 2000 Hz D. 1000 2000 3000 Hz 140. A young female patient with long history of sinusitis presents with frequent fever along with personality changes & headache of recent origin. The fundus examination revealed papilledema. The most likely diagnosis is : A. Frontal lobe abscess B. Meningitis C. Encephalitis D. Frontal bone osteomyelitis 141. Majority of cerebellopontine angle tumors are: A. Acoustic neuroma B. Meningioma C. Haemangioma D. Pontine Glioma 141. Nasopharyngeal angiofibroma arises from A. Posterolateral part of nasopharynx B. Vault of nasopharynx C. Posterior part of nasal cavity D. Pterygomaxillary fossa 142. Anterior ethmoidal sinus opens into A. Middle meatus B. Anterior turbinate C. Superior meatus D. Inferior meatus 143. Treatment of complete URT obstruction is A. Cricothyroidotomy B. Nasal intubation C. Needle in trachea D. Endotrachial intubation 144. Treatment of bilateral abductor vocal cord palsy is A. Adrenaline B. Tracheostomy C. Endotracheal intubation D. None of the above 145. Quantity of endolymph is A. B. C. D. 146. Korner's septum is formed by A. Petro-squamous suture B. Petro-squamous fissure C. Lateral margin of tegmen tympani D. Squamous tympanic fissure 147. External auditory canal is formed A. 1st branchial arch B. 2nd branchial cleft C. 1st branchial cleft D. 1st visceral cleft 148. After Type 1lI tympanoplasty, there is loss of all EXCEPT A. 15 db B. 25 db C. 30 db D. 5 db 149. After facial canal short process of incus, 3rd landmark for posterior tympanoplasty is A. Tympanic annulus B. Mastoid process C. Zygomatic ridge D. None of the above 150. Glomus jugular tumour is seen in A. Caroticotympanic B. Anterior tympanic membrane C. Hypotympanum D. Roof 151. Schwartze sign is seen in A. Otosclerosis B. Meniere's disease C. Glomustumour D. Glomus Jugulare 152. All are features of tympanic membrane rupture (printed oesophageal rupture in paper) EXCEPT A. Tinnitus B. Vertigo C. Conductive deafness D. Fullness in ear 153. Treatment of choice in atticoantral perforation is A. Surgery B. Medical treatment C. Medical & surgical treatment D. Radiotherapy 154. Rhinosporodiosis is caused by A. Protozoal infection B. Viral infection C. Bacterial infection D. Fungal infection 155. Treatment of choice in antrochoanal polyp is A. Surgery B. Medical C. Medical & surgical treatment D. Wait and watch 156. Eustachian tube opens into middle ear cavity at A. Anterior walls B. Hypotympanum C. Superior surface D. Posterior wall 157. Hyperacusis in belle's palsy is due to A. Stapedial paralysis B. Tensor tympani paralysis C. Tensor palati paralysis D. Trigeminal nerve paralysis 158. Cochlear implant used in A. Sensorineural deafness not improving with medical therapy B. Otosclerosis C. Meniere's disease D. Glomustumour 159. TRUE about Ramsay - hunt syndrome EXCEPT A. Involves VII nerve B. May involves VIII nerve C. Surgical removal gives excellent prognosis D. Causative agent is virus 160. Earliest manifestation of cerebellopontine angle tumor A. Loss of corneal reflex B. Ipsilateral lateral squint C. Ipsilateral tongue paralysis D. Ipsilateral pupillary dilation 161. Most common cause of stridor in neonates is A. Foreign body aspiration B. Laryngornalacia C. Pneumonia D. Epiglottitis 162. Most common cause of unilateral hearing loss is A. Measles B. Mumps C. Chickenpox D. Rubella 163. Most common extra-cranial complication of ASOM is A. Facial nerve paralysis B. Lateral sinus thrombosis C. Sub periosteal abscess D. Brain abscess 164. Rhinophyloma occurs due to A. Hypertrophy of meibobian gland B. Hypertrophy of the sebaceous gland C. Malignant tumour D. Papulo-pearly nodule 165. Hypertrophy of lingual tonsil occurs in A. Tonsillolith B. Tonsillectomy patient C. Tonsillar cyst D. Peritonsillar abscess 166. Which of the following is known as fourth turbinate A. Posterior ethmoid cells B. Anterior ethmoid cells (Aggar-nasi) C. Medial ethmoid cells D. Lateral ethmoid cells 167. Ciliary movement rate of nasal mucosa is A. 1-2 mm/min B. 2-5 mm/min C. 5-10 mm/min D. 10-12 mm/min 168. Most common sequence of ASOM is children A. Deafness B. Chronic-mastoiditis C. Cholestetoma D. Labyrinthitis 169. Which of the following part NOT included in hypopharynx is A. Pyriform sinus B. Post cricoid region C. Anterior pharyngeal wall D. Posterior pharyngeal wall 170. Which of the following region is involved in plummer-vinson syndrome A. Pyriform sinus B. Post cricoid region C. Valleculae D. Posterior pharyngeal wall 171. Motion sickness can be prevented by all EXCEPT A. Meteclopramide B. Domperidone C. Astimazole D. Cisapride 172. Semicircular canal involved in positional vertigo is A. Lateral B. Superior C. Inferior D. Posterior 173. Rinnes test will be negative if loss of hearing is A. 0-15dB B. 15-20dB C. 25-45dB D. 45-60dB 174. Lever ratio of tympanic membrane is A. 1.4-1 B. 1.3-1 C. 18.2-1 D. 1.5-1 175. All are tunning fork test EXCEPT A. Schwaback test B. Grants test C. Rinne's test D. Weber's test 176. Unilateral past pointing nystagmus is seen in A. Paralysis of posterior semcircular canal B. Paralysis of medial semicircular canal C. Follicular node lesion D. Cerebral hemisphere lesion 177. After sore throat patient develop acute sinusitis presents with all EXCEPT A. Diplopia B. Rhinorrhoea C. Headache D. Foul smelling 178. Allergic fungal sinusitis, true are all EXCEPT A. Tissue invasion is seen B. Endoscopic sinus treatment C. Enhancement C.T. scan is done D. Steroids are useful 179. Nasopharynx carcinoma is A. Basal cell carcinoma B. Squamous cell carcinoma C. Adenocarcinoma D. Epidermoid carcinoma 180. Radical neck dissection is done in all EXCEPT A. Supraglottic CA B. Glottic CA C. Transglottic CA D. Pyriform fossa tumour 181. 12 years child presented with discharge ear with deafness, on examination. Cholesteatoma & sensorineural deafness is present. Treatment of choice is A. Radical mastoidectomy B. Modified radical mostoidectomy C. Medical treatment D. Palliative surgery 182. In acoustic neuroma, cranial nerve to be involved earliest is: A. V B. VII C. X D. IX 183. A post dental treatment, presented with pain & swelling of SCM, examination reveals medially shift of the tonsil, Diagnosis is A. Parapharyngeal abscess B. Retrophoryngeal abscess C. Ludwing's angina D. Dental cyst 184. A child presented with biphasic stridor, treatment includes all EXCEPT A. Steroids B. Oxygen C. Radiotherapy D. Tracheostomy 185. "Pachydermia laryngitis" commonest site involvement is A. Inter arytenoid fold B. Posterior 1/3 & anterior 1/3 commisure C. Anterior 1/3 commissure D. Vestibular fold 186. Which of the following would be the most appropriate treatment for rehabilitation of a patient who has bilateral profound deafness following surgery for bilateral acoustic schwanoma: A. Bilateral high powered digital hearing AID B. Bilateral cochlear implants C. Unilateral cochlear implant D. Brain stem implant 187. True about CP angle tumour is: A. Absent corneal reflex B. Ipsilateral rectus palsy C. Pupillary dilatation D. Medial rectus palsy 188. Antro-choanal polyp are usually: A. Single and unilateral B. Multiple and bilateral C. Multiple and unilateral D. Single and bilateral 189. Main vascular supply of Little's area in all, Except: A. Septal branch of superior labial artery B. Nasal branch of sphenopalatine artery C. Anterior ethmoidal artery D. Palatal branch of sphenopalatine 190. Nasopharyngeal carcinoma presents as: A. Epistaxis B. Mass C. Headache D. Vertigo 191. White patch in the throat may be due to: A. Streptococcus B. Actinomycetes C. Sporotrichosis D. All of these 192. Injury to superior laryngeal nerve causes: A. Hoarseness B. Paralysis of vocal cords C. No effect D. Loss of timbre of voice 193. Most common location of vocal nodule: A. Anterior 1/3 and posterior 2/3 junction B. Anterior commisure C. Posterior 1/3 and anterior 2/3 junction D. Posterior commissure 194. A 31 year old female patient complains of bilateral impairment of hearing for the past 5 years. On examination tympanic membrane is normal and audiogram shows a bilateral conductive deafness. Acoustic reflexes are absent. All constitute part of treatment, except A. Hearing aid B. Stapedectomy C. Sodium fluoride D. Gentamycin 195. Commonest complication of CSOM is: A. Conductive deafness B. Meningitis C. Temporal lobe abscess D. Cholesteatoma 196. Patient presents with high fever, Signs of raised ICT and a past history of chronic otitis media likely diagnosis is: A. Brain abscess B. Pyogenic meningitis C. Acute subarachnoid hemorrhage D. Acute osteomyelitis of skull bone 197. The part most commonly involved in otosclerosis is: A. Oval window B. Round window C. Tympanic membranes D. Malleus 198. The most common cause of laryngeal stridor in a 60 years old: A. Nasopharyngeal carcinoma B. Thyroid carcinoma C. Foreign body aspiration D. Carcinoma larynx 199. Thudichum's nasal speculum is used to visualize: A. Anterior nasal cavity B. Posterior nares C. Tonsils D. Larynx 200. Pulsatile Tinnitus in ear is due to: A. Malignant otitis modes B. Osteoma C. Mastoid reservoir D. Glomus jugular tumour 1.Answer is B. •Unilateral damage-->present with voice change and hoarseness •Bilateral-->breathing difficulties and aphonia •Rt. recurrent laryngeal n. d/t relative medial location is more susceptible to damage during thyroid Sx •M/C cause of unilateral vocal cord palsy- ▪Idiopathic ▪Thyroid Sx •M/C cause of bilateral vocal cord palsy- Thyroidectomy •M/C cause of recurrent laryngeal nerve paralysis-- >bronchogenic carcinoma Superior Laryngeal Nerve Palsy Noticeable deviation of posterior commissure to paralyzed side during phonatory effort At rest, the vocal fold on paralyzed side is slightly shortened and bowed, and may be depressed below level of normal side. Loss of sensation to the supraglottic larynx can cause subtle symptoms such as frequent throat clearing, paroxysmal coughing, voice fatigue, vague foreign body sensations. Loss of motor function to cricothyroid muscle can cause a slight voice change, which the patient usually interprets as hoarseness. Most common finding is diplophonia (with decreased range of pitch, most noticeable when trying to sing 2. Answer is B. Catch-->irregular(other findings may be found in abscess also) . Clinical features vary according to the stage of the disease. Patients present with headache, fever, and otorrhea. The classic case of lateral sinus thrombosis in the preantibiotic era typically produced a picket fence fever curve, due to the periodic release of hemolytic streptococci from the septic sinus thrombus. With the occlusion of the lumen of the sinus, an interruption of the cortical venous circulation results in headache, papilledema, and increased intracranial pressure. Involvement of the torcular and sagittal sinus can result in otitic hydrocephalus. Tenderness and edema over the mastoid (the Griesinger sign) are highly suggestive of lateral sinus thrombosis and reflex thrombosis of mastoid emissary vein. With the extension of thrombophlebitis into the jugular bulb and internal jugular vein, pain may be present in the neck, particularly on rotation. Internal jugular vein may be palpated in the neck as a tender cord. The 9th, 10th, and 11th cranial nerve may be paralyzed by the presence and pressure of a clot in the jugular bulb (jugular foramen syndrome). Because the right transverse is usually dominant, the symptoms are more likely to occur when this sinus is involved. Recovery depends on the development of collateral circulation or possibly recanalization of the sinus. Because of this, the presence of anastomotic channel is important for recovery 3. Answer is B. Type A refers to eardrum movement within normal limits. Type B indicates little or no eardrum movement suggesting fluid in the middle ear space. A child with this type of tympanogram needs medical attention. Type C refers to a middle ear with negative pressure. Such a tympanogram may be caused by retraction of the eardrum or blockage of the Eustachian tube. Patients that present with Type As tympanograms normally have a history of middle ear problems ranging from childhood otitis media, otosclerosis and tympanosclerosis. TYPE Ad may present in patients who have had multiple perforations in the past, or multiple sets of grommets. Patients with his tympanogram may also have an ossicular chain disruption. (Rx-Ossiculoplasty) 4.Answer is C. This uses continuous and pulsed tone tracings. The normal graph recorded may be interleaved / continuous tracings below pulsed tone tracings. In patients with non-organic hearing loss will have opposite curves - their pulsed tracings are tracked below the continuous tracings. This type of curve is known as Type V Bekesy pattern. •Patient himself does the audiometry ▪I- N or conductive hearing loss ▪II- Cochlear ▪III/IV-Retrocochlear ▪V-Malingers 5.Answer is C. Remember the primary aim is to render the ear safe 6. Ans : B Croup is characterized by a "barking" cough, stridor, hoarseness, and difficult in breathing which usually worsens at night. The "barking" cough is often described as resembling the call of a seal or sea lion. The stridor is worsened by agitation or crying, and if it can be heard at rest, it may indicate critical narrowing of the airways. As croup worsens, stridor may decrease considerably. In radiology, the steeple sign is a radiologic sign found on a frontal neck radiograph where subglottic tracheal narrowing produces an inverted "V" shape within the trachea itself. The presence of the steeple sign supports a diagnosis of croup 7.Answer is A. Glue ear fluid is sterile. X-Ray of nasopharynx: Adenoid view 8. Ans: C 9. Ans: B This mnemonic helps when remembering the ascending order of structures that corresponds to each waveform in an auditory brainstem response (ABR) tracing: E COLI Mnemonic E - eighth nerve action potential (wave I) C - cochlear nucleus (wave II) O - olivary complex (superior) (wave III) L - laterallemniscus (wave IV) I - inferior colliculus (wave V) 10. Answer A Pain in the ear is referred from CN IX (peritonsillar abscess). CN X (cancer of the pyriform fossa) and CN V (ulcer tongue and temporomandibular joint). 11. Answer D Use of 2% acetic acid or dilure alcohol is the most-effective method to prevent otitis externa after swimming. Antibiotic, antifungal or steroid containing drugs are not recommended for prophylaxis. 12. Answer DAll the above Otitis media with effusion (OME) occurs due to defective eustachian tube function. All of the listed conditions affect function of the lube and cause OME. In a unilateral OME in an adult, always exclude carcinoma nasopharynx. 13.Answer is B. Any patient in cancer age group having persistent or gradual hoarseness of voice for >3 wks must have laryngeal examination to exclude cancer Clinical presentation and examination reveals a premalignant lesion of the vocal cords Rx: a.Cessation of smoking b.Micro laryngeal Sx for biopsy c.Regular follow up Best treatment at this stage is radiotherapy rather than bilateral cordectomy 14. Answer is A. • Gold standard for acoustic neuroma-->gadolinium MRI • Unilateral glue ear(adult)-->d/t nasopharyngeal Ca 15. Answer is B. •M/C cause of septal abscess-->septal hematoma •Always drain a septal hematoma as septal perforation is most common complication(since cartilage takes nutrition from perichondrium) 16. Answer is C. Superior orbital fissure syndrome- Infection of sphenoid sinus may affect structures of superior orbital fissure. Symptoms- 1. deep orbital pain 2. frontal headache 3. progressive paralysis of CN VI, III, IV in that order Orbital apex syndrome= Orbital fissure syndrome+ CN V- 2(i.e maxillary division) 17. Answer is D. • Elliptical space between vocal cords(Thyroaretenoid involved) • Triangular space in posterior glottis(Inter-aretenoid involved) • Key hole appearance of glottis(both muscle involved) 18. Answer is B. •Stapedial reflex- ◦Afferent- VIII N. ◦Efferent- VII N. 19. Answer D Since antibiotic treatment has been effective to relieve pain and fever, the child is left with a sterile fluid in the middle ear. If Ibis fluid does not drain spontaneously in another 12 weeks treat the case as one of otitis media with effusion. 20. Answer is C. • Choanal atresia is d/t persisting bucconasal membrane • McGovern's technique: Take an artificial nipple—> pierce the nipple—> baby breaths through nipple by sucking 21. Answer is B. Inspiration- Middle meatus Expiration- Middle meatus(mostly), inferior 22. Answer C In long-standing middle ear effusion, fibrous layer of tympanic membrane undergoes dissolution and as a result becomes thin and atrophic and easily amenable 10 form retraction pockets or cholesteatoma. Stasis of secretion also causes cholesterol granuloma in the mastoid. Ossicular necrosis rather than fixation is commonly seen. Long process of incus and stapes superstruclore undergo necrosis. 23. Answer C ) Mastoid swelling, fever and pulsatile ear discharge that the child has developed point to acute mastoiditis. Treatment would include IN antibiotics, analgesics and cortical mastoidectomy, Modified radical mastoidectomy is indicated in cholesteatoma. Antihistamines have no role. 24. Answer is A. • Children- Ethmoidal • Adults- Maxillary • Polyps-opposite of sinusitis ◦ Children-Maxillary ◦ Adults-Ethmoids • Carcinoma-Maxillary • Mucocoele-Frontal-->X-Ray-->loss of scallops 25. Answer is A. Nerve supply of Parotid- • Sensory- Auriculotemporal • Secretomotor- Glossopharyngeal 26. Answer B Patulous eustachian tube is an abnormally patent tube seen in the third trimester of pregnancy or in rapid weight loss. Symptoms complained by her are due to autophony. Conductive hearing loss due to otosclerosis, otitis media with effusion and retracted tympanic membrane also conduct body sounds to the ear but her tympanic membrane findings of movement with respiration are typically seen in patulous eustachian tube. 27. Answer is B. Optic-->Superolateral Internal carotid-->Lateral 28. Answer A Glomustumour is five times more common in women. It is a very slow-growing tumour. Sometimes glomustumour does not occur alone but is associated with chemureceptortumours, seen in carotid body and on opposite side of the body signifying its multicentric origin. These tumours are extremely vascular and bleed profusely; diagnostic biopsy is therefore contraindicated. Diagnosis is made by high resolution gadolinium enhanced CT. 29. Answer is B. 2-->formed due to sup. Semicircular canal 3--> thin plate of bone separating cranial and tympanic cavities 30. Answer is A. to rule-out nasopharyngeal carcinoma 31. Answer is C. Never changed-->Usually kept < 48 hours (when> 48 hrs-->give antibiotic coverage) 32. Answer D Griesinger's sign is oedema over the mastoid due to thrombosis of mastoid emissary vein. It is seen in lateral sinus thrombosis. Other three are features of glomustumour when it arises from dome of jugular bulb (glomusjugulare). It produces red flush in the lower pan oftympanic membrane resembling a "rising sun", Brown's signis elicited with pneumatic otoscope when pressure is raised in the ear canal,tumour pulsates vigorously and when pressure is further raised pulsations stop altogether and tumour blanches. Reverse occurs on decreasing the pressure. It is also called "pulsation Sign". Aquino's sign is blanching of tumour on compression of ipsilateral carotid. 33. Answer is B. • Preauricular sinus-->If arch fusion is not complete • Callover fistula-->it is the 1st branchial cleft anomaly • Second bronchial fistula- ◦Sternocleidomastoid(Upper 1/3)-->internal opening--> near tonsils ◦ In close proximity to 2nd bronchial fistula--> Carotid A.(int./ext.), hypoglossal nerve 34. Answer is D. • External ear is supplied by- ◦ Auriculotemporal nerve(V3) ◦ Greater auricular nerve(C-2,3) ◦ CN VII & X ◦Lesser occipital nerve (C-2) 35. Answer is C. Bony landmark(Anatomical/Surgical)-->McEvan's triangle(Suprameatal triangle) 36. Answer is B. Tragal sign--> Furunculosis 37. Answer is B. • When does the canal completes it's development is the question Clinical Importance • BAHA- Bone Anchoring Hearing Aid given upto 5-yrs • Ear surgery for atresia-->5-7 yrs-->when canal development is full 38. Answer A For a smalltumour confined to promontory, surgical excision is the treatment of choice. Embolisation reduces vascularity of the tumour and decreases its size. It is used either pre-operatively before surgery or is the sole treatment in inoperable patients who have received radiation. Radiation therapy reduces the vascularity of tumour and arrests its growth. It is used in elderly patients or inoperable large tumours, residual tumoursortumours that have recurred after surgery. 39. Answer is B. (Adult-->12-15 mm) 40. Answer is C. (1.25 cm below and behind) 41. Answer is A. (base-->high frequency) 42. Answer D Paraganglioma (glomustumour) arises from the paraganglionic tissue normally present in the adventitia of dome of jugular bulb, glomus body on the promontory of middle ear, at the carotid bifurcation and glomus body along the vagus nerve at the base of skull. They have been called glomustympanicum, glomusjugulare, carotid body tumourandglomusintravagale, respectively. Most common tumours of facial nerve are neuroma and haemangioma. 43. Answer is B. (it is for middle ear Sx) Special finding in petrositis-->Pulsatile discharge(also in ASOM) •Pulsatile tinnitis-->Glomus tumor •Fluctuating tinnitis-->Meniere's disease 44. Answer is B. M/C-->Posterosuperior quadrant 45. Answer is C 3-->when it progresses Meneier's-->false positive 46. Answer is B. (blanching of tumor on siegel's speculum) 47. Answer is D. (either gets blocked or dead) 48.Answer is A. •Canasopharynx is most commonly seen in fossa of Rosenmuller •This fossa is just behind the Eustachian tube and hence a Ca of this fossa often blocks the tube leading to Glue ear •A patient of glue ear will present with dull TM, deafness and tinnitus, type B curve impedence. The most common presentation of Canasopharynx is neck mass. 49. Answer is B. (CROS-->Contralateral Routing of signals) Normally both ears are needed for spatial orientation 50. Answer is C. • Post. Auricular abscess- commonest, abscess forms over mastoid, pinna is displaced forwards. • Bezold's abscess- abscess deep to sternomastoid • Cittelli abscess- Pus breaks through inner table of mastoid tip and travels along the posterior belly of diagastric muscle. Swelling is seen in the diagastric triangle of neck. 51. Answer is B. • Superior or post. superior (P=79) • Otoscopy finding-->hanging superior canal wall 52. Answer is C. I/II Radiotherapy III/IV Sx and radiotherapy Keratinization-->Stripping 53. Answer is C. • At the junction of-->anterior 1/3 and posterior 2/3 • Early-->Pinkish pale, mobile mucosa-->cured by voice rest • Late-->White or gray, fixed mucosa--> voice rest-- >no improvement-->Sx 54. Answer is C. • Aspregillusfumigatus-->non-invasive-->Forms fungal ball(Aspergilloma) • Mucormycosis-->dangerous-->since it is angioinvasive 55. Answer D CSOM is notorious {or intracranial complications. The first priority in mastoid surgery is given to free drainage of pus to render the ear safe. Second priority is given to preserve or reconstruct hearing mechanism. 56. Answer is B. COWS (Dhingra-42) Hyperactive-->Warm water like action hypoactive-->Cold water like action 57. Answer is B. Acoustic neuroma--> MRI--> Gold standard (P=110) Angiofibroma Clinical Diagnosis (Dhingra-231) Nasopharyngeal Ca MRI+Gadolinium (Dhingra-235) 58. Answer A Otosclerosis mostly involves females, has a positive family history and causes slow progressive bilateral hearing loss. Audiogram is characterised by Carhart'snoteh-a dip in booe conduction at 2000 Hz. 59. Answer B Caucasians have highest prevalence. Japanese and Chinese have low incidence. Lowest incidence occurs in African Negroes. 60. Answer is B Accessory maxillary sinus ostium is located in the lateral wall of the middle meatus, usually in the anterior or posterior fontanelle where the medial wall of the maxillary sinus is membranous. 61. Answer is D (may go in brain-->then segmental resection of brain) 62. Answer is B • M/C indication of tracheostomy: Ca larynx • Only indication for high tracheostomy: Ca larynx • M/C source of bleeding in tracheostomy- Inf. Thyroid vein • M/C complication of tracheostomy- ▪ Hemorrhage ▪in children-->Difficult decannulation ▪in adults--> Stenosis Gold standard Rx of stenosis--> incision + end to end stenosis • Structures damaged during tracheostomy- ▪ Isthmus ▪ Inf. Thyroid vein ▪ Thyroidea ima artery • Only advantage of metallic tube in tracheostomy--> is in cleaning and changing • How to clean non-metallic tube--> Instill Sod. Bicarb.--> then suck • Shiley's/Koenig's tube—>Paediatric • Montogmery--> T-Tube(Silicon) Cricothyroidotomy and percutaneous tracheal ventilation are preferred over tracheostomy in most emergency situations. Disadvantage in tracheostomy is the inability to place a tube of greater than 6 mm in diameter due to the limited aperture of the cricothyroid space. It is also relatively contraindicated in patients under 12 yrs of age because of risk of damage to cricoid cartilage and subsequent risk of subglottic stenosis. 63. Answer A Otosclerosis is autosomal dominant disease. 64. Answer is A. Reduce size and + fibrosis • But RT not used routinely-->since boy is juveline and radiation increases bone Cancer • Normal Rx of angiofibroma-->Sx 65. Answer is B. Parts of temporal bone- (MTP-SS) • Mastoid • Tympanic • Petrous • Squamous • Styloid process 66. Answer is 3. Otosclerosis is a d/e of bony labyrinth(otic capsule) 67. Answer A Most common site forotosclerosis to start is fissulaante- fenestraman area lying in front of oval window. Other sites listed in the question are also affected but their involvement is less frequent 68. Answer is D. More than 100 indications-->so go for all(always except like obvious-appendectomy) 69. Answer is C. • SEE STEROIDS ARE USUALLY C/I IN FUNGAL INFECTIONS • M/C cause of allergic sinusitis--> House dust 70.Answer is D. Cochlear implants are the electronic devices which convert the sound signal into the electric impulses that directy stimulate cochlear nerve 71. Answer is D. Ask EEE (adduction) and not AAA(Abduction)--> then anterior commisure is seen (usually difficult to see in indirect laryngoscopy) 72. Answer is B. (terbinate=concha) 73. Answer is A. • Sciebe's is the commonest aplasia • Congenital deafness--> U-Shaped audiogram 74. Answer C Otic capsule (also called bony labyrinth) has three layers: outer periosteal, inner endosteal and the middle bony. The bony layer which develops from cartilage and for some unknown reason bone of the otic capsule is absorbed and replaced by spongy vascular bone which later becomes sclerotic due 10 deposition of calcium. 75. Answer C Tympanometry in stapedialotosclerosis shows AS curve. AD curve is seen in ossicular discontinuity. Slowly progressive conductive hearing loss in an adult with normal tympanic membrane in appearance and mobility, and normal function of eustachian tube, is typical of otosclerosis. A Normal As Reduced compliance at ambient pressure (otosclerosis) AD Increased compliance at ambient pressure (ossiculardiscontinuity) B Flat or dome shaped (fluid in middle ear) C Maximum compliance at pressures more than -100 mm H2O (negative pressure in middle ear) 76. Answer is C. Type I- Myringoplasty Type II- No Name Type III- Columella Type IV- No Name Type V- Fenestration operation(fistula in horizontal SCC-- >vertigo) 77. Answer is B. (P-100 is not used now) 78. Answer D Schwartze's sign is reddish hue seen over the promontory. It is due to increased vascularity of bone and is seen in early or active otospongiosis. Reddish hue seen through intact tympanic membrane in its lower part is due to glomusjugularetumour and is sometimes also referred to as "rising-sun appearance". Better hearing ability in noisy surroundings is a feature of otospongiosis and is called paracusisWillisiana or paracusis of Willisi. 79. Answer is A. • Lymphoepithelioma- ◦ M/C Site- nasopharynx ◦ in fact most common type of nasopharyngeal Ca (in WHO classification) ◦ Rx- RT Woodworkers- Adeno-->highly radioresistant-->Rx-->Sx 80. Answer is B. chronic persistent positional vertigo--> singular nerve neurectomy 81. Answer is A. Active stage-->NaF-->then Sx 82. Answer is D. (P=340) 83. Answer is A. HOLMAN MILLER SIGN (ANTRAL SIGN)- The anterior bowing of the posterior wall of the antrum seen on lateral skull film. Pathognomic for juvenile nasopharyngeal angiofibroma 84. Answer B A negative Rinne's test for 256, 512and 1024 Hz will show a minimum AB gap of 15, 30 and 45 dB, respectively. 85. Answer D Though A, B and C, are features of Van der Hoeve syndrome, preauricular sinuses are not associated with this syndrome. 86. Answer B Carhart's notch is loss of bone conduction in audiogram. It is a feature of otosclerosis. Loss s is 5dB in 500Hz 10 dB in 1000 Hz 15 dB in 2000 Hz 5 dB in 4000 Hz 87. Answer D Carhart's notch is a sensorineural hearing loss centred maximally at 2 kHz in bone conduction curve of the audiogram. It is only an apparent loss due to stapes fixation and can be reversed by stapedectomy. 88. Answer B Stapedectomyor stapedotomywith a prosthetic connection between incus and oval window is the treatment of choice. Hearing aid can also be useful if patient refuses surgery. Stapes mobilisation will also restore hearing but the benefit is short-lived due to refixation. Fenestration operation in which a fenestra is made in the horizontal canal and covered with a tympanomeatal flap has been abandoned as this leaves a mastoid cavity which requires lifelong aftercare and also a hearing loss of 25 dB. After stapedectomy there is potential risk of giddiness, therefore it is not advised to high construction workers and divers. It is also contraindicated in professions in which patient has to strain or frequent pressure changes occur in the middle ear. e.g. professional athletes or frequent air travellers. In them there are chances of developing perilymph fistula with consequent vertigo and sensorineural hearing loss. Since the person in question is an office-goer, it does not form a contraindication to stapedectomy. 89. Answer is 2. • Incus (lenticular part most common) • M/C Site: IS joint(towards incus side) Galle's test: -ve condition-->Otosclerosis, Ossicular Chain disruption 90. Answer C Tullio phenomenon is loud sounds or noise producing giddiness. It is seen in congenital syphilis, Meniere's disease and when three functioning windows are present in the ear, i.e, round window, oval window and a third artificially created window such as fenestration of the lateral canal (an old operation for otosclerosis) or a fistula on semicircular canal. In Meniere's disease a distended saccule lies against stapes footplate and in congenital syphilis adhesions form between membranous labyrinth and the footplate. Thus movement of stapes due to loud sounds stimulates labyrinth in both these conditions. ParacusisWillisiis seen in otoscleroties. They hear better in noisy than in quiet surroundings. Hennebert'ssign.It is a positive fistula test without the presence of a fistula. It is seen in congenital syphilis or Meniere's disease (25% cases). Otolithic crisis of Tumarkin or drop attacks are seen in early or late Meniere's disease, Patient feels as if pushed to the ground without any vertigo or loss of consciousness. It is presumed they are due to distortion of otolithic membrane of utricle or saccule when endolymphatic pressure rises. 91. Answer D In the above question, patient is a female with bilateral conductive hearing loss with normal tympanic membrane. Also the As curve indicates that compliance of tympanic membrane is restricted indicating ossicular fixation. Absence of stapedial reflex indicates that contraction of stapedius muscle is not producing any change in compliance of 1M again indicating stapediallixation. The diagnosis points to otosclerosis. Treatment of otosclerosis includes A, B and C. Gentamicin is never used. Gentamicin perfusion of the middle ear is used in Meniere's disease to selectively destroy vestibular end organs and to stop giddy attacks. 92. Answer is A. Rt ear-->Rt hand; Lt ear-->Lt hand Indirect laryngoscopy-->Left hand used (always) 93. Answer is B. (occurs d/t smoking) 94. Answer B All of the above conditions cause conductive deafness. Otosclerosis usually starts between 20 and 30 years. It is rare before 10 years. Acute otitis media is usually unilatera1 and presents with severe pain in the ear. Congenital cholesteatoma is rare condition and extremely rare to be bilateral. OME is the most common cause affecting children. 95. Answer D Acoustic neuroma and endolymphichydrops (Meniere's syndrome) present with tinnitus, dizziness and progressive hearing loss. This also holds true for meningiomas which involve cerebellopontine (CP) angle. Nearly 10% of tumours of CP angle are meningiomas. Histiocytosis X. presenlly called Langerhans cell histiocytosis, is a rare disease which may involve temporal bone (as eosinophilic granuloma) or multiorgan disorder involving skull, long bones, ribs. Vertebrae, pelvis, maxilla and mandible, and other non-osseous organs. Langerhans cells are involved in cell-mediated immunity, cause osteolytic lesions in bones and recruit eosinophils. Bosioophilic granuloma is a milder form of Langerbans cell histiocytosis which can involve temporal bone. 96. Answer is C. • Tonsil--> deep lobe involved--> Total parotidectomy • Usually superficial lobe is involved--> Superficial dissection 97. Answer is B. (least imp, also benign) 98. Answer is B. (most commonly injured) • Parotid gland-->Stenson's duct • Submandibular gland--> Wharton's duct 99. Answer C The patient suffers from sensorineural deafness, so fenestration and stapes mobilisation cannot be done. Such patients require hearing aid or cochlear implant. Since hearing aid has not shown any benefit, cochlear implant will be indicated to provide hearing and develop speech and language, There is no conservative treatment. 100. Answer is C. Medial geniculate body of thalamus-->Perception possible Frequency coding- • Cochlea- ◦ Apical- ◦ Basal- low frequency(ck) 101. Answer D Tympanic membrane develops from all the three germinal layers. Outer epithelial layer develops from epithelium lining the first branchial cleft Inner mucosal layer develops from the lining of tubotympanic recess which is a derivative of first pharyngeal pouch and partly of the second pouch. Intermediate fibrous layer is derived from mesoderm. 102. Answer D Bony labyrinth, also called the otic capsule, develops from cartilage which later ossifies to form bone. There are 14 centres of ossification. Ossification starts at 16th week when first centre appears near the cochlea. The last centre appears at 20th week at the posterolateral part of the posterior semicircular canal. 103. Answer C ) Lateral surface of tympanic membrane is supplied by auriculotemporal nerve in its anterior half and auricular branch of vagus in the posterior half. The medial surface in its entirety is supplied by CN IX through its tympanic branch. 104. Answer B Round windowis covered by secondary tympanic membrane. Stapedius muscle arises within the hollow body of pyramid and stapedial tendon comes out of its tip to get attached to neck of stapes. Sinus tympani is an open deep depression in the medial wall of middle ear. It lies medial to vertical part offacial nerve and the pyramid. Above it is bounded by ponliculus-a bony bar connecting pyramid to promontory-and below by the subiculum. 105. Answer B Though pinna has attained the adult configuration (shape) by birth, it continues to grow in size after birth and attains 90-95% of the adult size by the age 5-6 years. This is the age when plastic surgical correction of pinna can be done. 106. Answer C Glossopharyngeal nerve (CN IX) gives a branch called Jacobson's nerve (tympanic branch of glossopharyngeal) which along with caroticotympanic nerves fonns tympanic plexus on the promontory of middle ear. Glossopharyngeal nerve is sensory while caroticotympanic nerves carry sympathetic fibres. Tympanic plexus supplies innervation to the medial surface of tympanic membrane. tympanic cavity, mastoid air cells and bony eustachian tube. 107. Answer C Sinodural angle is also called the Citelli's angle. It is formed at the site where dura of middle cranial fossa meets the dura of sigmoid sinus. It forms an important landmark in mastoid surgery. McEwen's triangle is bounded by temporal line, posterosuperior segment of the external auditory canal and a line drawn as a tangent 10 the external canal. It is an important landmark to open the mastoid anl111lll-an initial step for mastoidectomy. Solid angle is the place where three semicircular canals meer. 108. Answer A ) Total length of external auditory canal from concha to tympanic membrane is 24 mm (compare eustachian tube which is 36 mm), Its outer one-third is cartilaginous while inner two-third are bony. Hair follicles, sebaceous and cerumInous glands are confined only to its outer one-third. Boil. a staphylococcal infection of the hair follicle, therefore occurs only in the outer part of the canal. Floor of the cartilage forming outer cartilaginous canal may show dehiscences called fissures of Santorini, which permit infections of the parotid or from the mastoid to present in the canal and vice versa. 109. Answer A Komer's septum is a bony plate, sometimes present in the mastoid, dividing superficial squamous cells from the deeper petrosal cells. Antrum lies deep to it Antrum cannot be located unless this septum is removed during mastoid exploration, Facial recess lies lateral to sinus tympani. Middle ear can be entered through the facial recess in combined- approach tympanoplasty or surgery for cochlear implant. Trautmann’s triangle is· a plate of bone between solid. Angle, sinodural angle and sigmoid sinus (Fig. 1.2). It is a landmark for entry into the posterior crania1 fossa. Arcuate eminence is seen in superior surface of petrous bone. Underneath this eminence lies the superior semicircular canal. 110. Answer D Normal tympanic membrane shows mobility on siegalisation. These movements are lost or, restricted in middle ear effusion or when tympanic membrane is thick or markedly retracted. The tympanic membrane becomes congested as a result of crying, sneezing orblowingofnose.Itis common to see a congested tympanic membrane in a crying child. In a bulging tympanic membrane lateral process of malleus audits handle maybe obscured. 111. Answer C The nerve trunk lies 6-8 mm deep to tympanomastoidsuture, It is the most reliable landmark 112. Answer C Endolymph is secreted by striavascularis which forms the outer wall of scala media (also ca1led the cochlear duct). It is absorbed through endolymphatic sac. 113. Answer C In radical mastoidectomy, middle ear is exteriorised into extemal ear cana1 and the eustachian tube is blocked by packing it with muscle or cartilage to prevent nasopharyngeal infections reaching the middle ear. 114. Answer B Stapes superstructure, i.e. its head, neck And both crura develop from Reicben'seartilage (second arch). Stapes footplate has a dual origin both from otic capsule and second arch. 115. Answer A Due to changes in maturation of central nervous system, findings of BERA change during infancy. Therefore, only age specific latency and intensity findings are used to interpret results. However, it is not affected by sleep, sedation or attention of the child. In fact sedation is used to elicit responses in infants and children. 116. Answer E Radiation, rubella (German measles), thalidomide administration or infection with cytomegalovirus during pregnancy have a teratogenic effect on the development of inner ear particularly in the first trimester, 117. Answer C Studies in experimental animals and human temporal bones have shown oedema of striavascularis and reduced blood supply to lateral wall of cochlea, This action on striavascularis results in reduction in endocochlear potentials and thus cause rise in threshold of compound action potential. Furosemide causes both temporary and permanent hearing loss. 118. Answer A Jacobson's nerve is the other name for tympanic branch of glossopharyngeal nerve (CN IX). It supplies sensory innervation to mucosa of the middle ear and mastoid and also carries preganglionic parasympathetic secretomotorfibres to the parotid gland via lite lesser petrosal nerve and otic ganglion. Section of these fibres interrupts secretomotor nerve supply to the parotid and thus helps in the relief of gustatory sweating (Frey's syndrome). 119. Answer C Normally in a sitting position, lateral canal is tilted 300 backwards. By extending the bead 60° backward, canal assumes a vertical position which is ideal for caloric test. 120. Answer B Dorello'scannltransmits CN VI. It lies between petrous apex laterally and pelroclinoid ligament superornedially. It is involved in petroSitistheGnldenigo's sign. 121. Answer D Costen's syndrome is due to temporomandibular (TM) joint abnormality with defective bite. It causes pain in the ear and the surrounding frontal, parietal and occipital regions, along with vertigo and tinnitus. It is treated by analgesics, local heat and slow exercises of TM joint. Defective bite can be corrected by an orthodontist. 122. Answer A Coalescent mastoiditis is treated by cortical mastoidectomy, also known as Schwartz operation. In this operation all mastoid air cells are exenterated leaving posterior canal wall intact. 123. Answer C CN X (vagus) nerve supplies both the ear and respiratory tract and lungs. Manipulation of ear canal such as removal of wax from the canal provokes cough. 124. Answer B Negative Rinne on the right means bone conduction better than air conduction, meaning thereby conductive deafness. But this test has been done without masking the left ear which is normal in this case. Response to bone conduction could be from the left ear. Thus right ear suffers from sensorineural deafness. Rinne's test in this case is "false negative". 125. Answer B Character of tinnitus in Meniere's disease is roaring or hissing type. Pulsatile tinnitus (non-continuous tinnitus) is due to vascular or nonvascular causes. Common aetiologies include: (i) Raised intracranial pressure including benign intracranial hypertension (ii) Glomustumours (iii) Hypertension (iv) Venous hum (v) High jugular bulb (vi) Atherosclerotic carotid artery disease (vii) Arteriovenous fistula or malformation (viii) Thyrotoxicosis, pregnancy due to increased cardiac output (ix) PersIstentstapedial artery (rare) (x) Palatal myoclonus or myoclonus of tensor tympani or stapedius muscle (non-vascular). 126. Answer C Malignant otitis extema is not a neoplastic disease. The term malignant is used because of lhe severity of disease which can cause several complications including multiple cranial nerve palsies. Malignant otitis externa is an infective process caused mostly by P. aeruginosa. II affects elderly diabetics or Ibose with immunocompromised status. 127. Answer C Treatment of glue ear where thick secretions accumulate in middle ear would be drainage through a myringotomy and long-term aeration of middle ear through a grommet. Options B and D are only myringotomy which will soon close leading to recurrence of disease. 128. Answer B Facial nerve palsy occurs in 50% of transverse and less than 20% of longitudinal fractures. Palsy is not always due to transection of nerve. It C&1 occur will the formation of intraneuralhaematoma or a spicule of bone embedded iii the nerve. CSF otorrhoea does not occur in all cases of fractures. 129. Answer B The option B should have been primary cholesteatoma and not secondary. Secondary cholesteatoma occurs due to migration of squamous epithelium through a preexisting perforation such as posterosuperior marginal or total perforation (as occurs after acute necrotising otitis media). There is no term like tertiary cholesteatoma as given in option D. 130. Answer C Aim of surgery in middle ear infections is to make the ear safe. Since attic cholesteatoma in Ibis case is complicated by lateral sinus thrombosis, modified radical mastoidectomy (canal wall down procedure) will be justified. In this case, the disease will be exteriorised so that the cavity could be subsequently examined and cleaned if cholesteatoma was left behind or recurs. Intact canaI wall mastoidectomy and simple mastoidectomy are "canal wall up" procedures. Obliteration of mastoid cavity even after removal of cholesteatoma runs the risk of burying cholesteatoma and subsequent complications. 131. Answer B Impedance audiometry is not used in children before 4 months of age because of the characteristics of ear canal. Ear canal is collapsible and a true tympanogram is difficult to get. Similarly behavioural audiometry and free field audiometry arc not possible in a neonate. They cannot test the level of hearing. Results of behavioural audiometry will vary due to subjective observation. The only reliable method is brain-stem evoked response audiometry (BERA). In this air conducted clicks arc presented through headphones and response picked up from surface electrodes applied to scalp. They arc also not affected by sleep and sedation. Latency and morphology of BERA waves changes rapidly with age in infancy and therefore only age specific norms of latency and amplitude are used to interpret results. 132. Answer A Note that ossicular disruption with intact tympanic membrane (TM) causes more hearing loss than ossicular disruption with perforated tympanic membrane. Hearing loss caused by different lesions of conducting apparatus is as follows: (i) Complete obstruction of ear canal: 30 dB (ii) Ossicular disruption with intact TM: 54 dB (iii) Ossicular disruption with TM perforation: 38 dB (iv)Perforation of TM: Varies from 10 to 40 dB depending on size and site of perforation (v)Otitis media with effusion (fluctuating hearing loss): 2040 dB (vi) Complete closure of oval window: 60 dB 133. Answer C A Siegel's speculum bas a magnifying glass and pump attached to it.The pump helps 10 increase or decrease air pressure in the ear canal while lens gives magnification. If a powder like antibiotic or antiseptic (iodine plus boric acid) is placed in the speculum, it can be blown in the ear for treatment of otitis media or otitis externa. A Siegel's speculum is used in the following conditions: (i) To magnify tympanic membrane to see small perforations. (ii') To see mobility of tympanic membrane. (iii) To perform fistula lest. (iv) To insufflate powder into the ear canal and middle ear. 134. Answer A Gradenigo's syndrome classically consists of a triad of (i) deep-seated or retro-orbital pain, (ii) diplopia due 10 CN VI involvement,and (iii) otorrhoea due to middle ear or mastoid infection. II is seen in petrositis where infection from the middle ear and mastoid spreads to the petrous apex with formation of extradural abscess. II may be associated with several other symptoms such as earache, facial paralysis, acoustic or vestibular symptoms, or hearing loss, but they are not included in the syndrome. 135. Answer C In Weber's test, tuning fork is placed on the forehead or vertex in the midline. Sound is conducted equally 10 both ears through bone vibration. In a normal person.the sound is centralised as it is beard equally in both ears. In conductive deafness, sound is localised 10 the affected ear (in this case right ear) because cochlear function is normal. The cochlear function is normal in healthy ear also but the ambient noise of air-conducted sound produces a masking effect on the normal side. Masking effect of air- conducted sound on the diseased ear (right ear in this case) is missing. 136. Answer B Bilateral hearing loss getting worse in pregnancy is due 10 otosclerosis. Tympanogram in this case would be A. Iype. It is a normal Iympanogram with reduced compliance due 10 stapes fixation. 137. Answer A Pure tone and speech audiograms are measured in dB HL (hearing level). In normal bearing adult person bearing threshold is 0 dB HL. dB SL (sensation level) is the intensity of sound that will produce same sensation of hearing as in a normal person. Thus in a hearing impaired individual who already has a hearing loss of 20 dB, a sound of 50 dB will produce a sensation of 30 dB only. dB SPL (sound pressure level) is the physical measure of sound intensity. Environmental sounds (noise pollution levels) are measured in dB A. dB SHL is used in auditory brain-stem response to click stimulus. 138. Answer B Otoacoustic emissions (OAEs) are produced by the outer hair cells. Motility of outer hair cells, spontaneous or in response to sounds, is transmitted by the basilar membrane, ossicular chain to tympanic membrane, and can be picked up and measured by a sensitive microphone placed in the outer ear canal. Spontaneous OAEs are present in 70% of normal individuals. Evoked OABs are produced in response of pure tones or clicks. Absence of OABs indicates damage to outer hair cells as in ototoxicity. 139. Answer C 500, 1000 and 2000 Hz are the speech frequencies. Average ofthreshold levels at these frequencies is called pure lone average (PTA). PTA should agree with± 7 dB of speech reception threshold (SRT). IfSRTisunusually better relative to PTA, it indicates non-organic hearing loss or malingering. PTA is also used to measure percentage of hearing impairment in one or both ears and in measurement of the hearing handicap. 140. Answer is A 141. Answer is C. (Posterior part of the nasal cavity): Nasopharyngeal Fibroma (Juvenile nasopharyngeal angiofibroma) Origin - i) Arise from the posterior part of the nasal cavity close to the superior margin of sphenopalatine foreman. ii) Usually arise from the lateral wall of the nasopharynx (III-LT)  Commonest benign tumour of the nasopharynx, young boy (10-20 years age)  Profuse and recurrent episistaxis is the most common presentation  Progressive nasal obstruction and de-nasal speech  Conductive hearing loss and serous otitis media  Mass in the nasopharynx - sessile, lobulated or smooth and obstructs one or both choane pink or purplish in colour.  Consistency is firm but digital palpation should never be done until at the time of operation. Other clinical features like broadening of nasal bridge. proptosis. swelling of cheek, infratemporal fossa or involvement of II, III, IV, VI, cranial nerve depend on the extent of tumour  * Biopsy is avoided (contraindicated)  Investigation of choice - CT scan with enhancement  Surgical excision is now the treatment of choice. 142. Answer is A. (Middle meatus): Openings in meatus I. Superior meatus II. Middle meatus III. Inferior meatus Sphene-ethmoidal recess Posterior ethmoidal sinus Frontal air sinus Maxillary air sinus Middle (Anterior) ethmoidal sinus (Dacrocystorrhinostomy openings) Nasolacrimal duct Sphenoidal air sinus 143. Answer is C. (Needle in trachea): In emergency situation where facilities are not present needle in the trachea is best option, otherwise, tracheostomy should be done. 144. Answer is B. (Tracheostomy): Indications for tracheostomy A. Respiration obstruction 1. Infections  Acute laryngo-tracheo-brcnchitis, acute epiglottis, diptheria - Ludwig's angina, peritonsilar, retropharyngeal or parapharyngeal abscess, tongue abscess. 2. Trauma  External injury of larynx and trachea  Trauma due to endoscopies, especially in infants and children  Fractures of mandible or maxillofacial injuries 3. Neoplasms - Benign & malignant neoplasms of larynx, pharynx, upper trachea, tongue and thyroid. 4. Foreign body larynx 5. Oedema larynx due to steam, irritant fumes or gasses, allergy (angioneurotic or drug sensitivity), radiation 6. Bilateral abductor paralysis 7. Congenital anomalies - Laryngeal web, cysts, tracheo-oesophageal fistula - Bilateral choanal atresia 145. Answer is B. (150 l): Origin Absorption Perilymph (i) From CSF (ii) Direct blood filtrate from the vessels of spiral ligament Endolymph (i) Secreted by stria vascularis or by the adjacent tissue of outer sulcus (ii) Derived from perilymph across Reissner's membranes Through aqueduct of cochlea in subarachnoid space Saccusendolyrnphaticus Stria vascularis 146. Answer is A. (Petro-squamous suture):  Mastoid develops from the squamous and petrous bones. The petro squamosal suture may persist as a bony plate the Korner's septum, separating superficial squamous cells from the deep petrosal cells.  Korner's septum is surgically important as it may cause difficulty in locating the antrum and the deeper cells, and thus lead to incomplete removal of disease at mastoidectomy.  Mastoid antrum cannot be reached unless the Korner's septum has been removed. 147. Answer is C. (1 st branchial cleft):  External auditory meatus - develops from the first branchial cleft by about the 16 th embryonic week.  External ear canal is fully formed by the 28 th week.  Auricle: First branchial cleft is the precursor of external auditory canal. Around the axis. 148. Answer is C. (30 db): TYMPANOPLASTY - reconstruction of sound conducting mechanism of middle ear. Type I Type II Type III Type IV Type IV Defect is perforation of tympanic membrane which is repaired with graft. It is also called myringoplasty. Defect is perforation of tympanic membrane with erosion of malleus. Graft is placed on the incus or remnant of malleus Malleus and incus are absent. Graft is placed directly on the stapes head is placed directly on the stapes head. It is also called myringostapedioxy or colummellatympanoplasty Only the foot plate of stapes is present Stapes foot plate is fixed but round window is functioning. In such cases another window is created on horizontal semicircular canal and covered with a graft. Also called fenestration operation 149. Answer is A. (Tympanic annulus): Landmark for posterior tympanoplasty  1 st landmark - Facial canal  2 nd landmark - Short process of incus  3 rd landmark - Tympanic annulus 150. Answer is C. (Hypotympanum): GLOMUS TUMOUR - most common benign neoplasm of middle ear origin from the glomus bodies (consists of paraganglionic cells derived from the neural crest) • Common in middle age (40-50 years). Female preponderance. (i) Glomus jugulare - They arise from the dome of jugular bulb, invade the hypotympanum and jugular foramen, causing neurological signs of IX to XII cranial nerve involvements. They may compress jugular vein or invade its lumen. (ii) Glomus tympanicum - They arise from the promontory of the middle ear and cause aural symptoms with facial paralysis. 151. Answer is A. (Otosclerosis):  "Schwartz sign" - is a pink reflex, seen through intact tympanic membrane in the area of oval window, It indicates active otosclerosis usually during pregnancy.  "Griesinger's sign" - is seen in lateral sinus thrombosis. It is due to thrombosis of mastoid emissary vein impending venous drainage and thus causing oedema over the mastoid  "Thumb sign" - Acute epiglottitis  "Steeple sign" - Acute /aryngotracheobronchitis 152. Answer is C. (Conductive deafness): Traumatic rupture of Tympanic membrane a. Trauma due to hair pin, match stick or unskilled attempts to remove a foreign body b. Sudden change in air pressure ego a slap or kiss on the ear or a sudden blast, Forceful valvalsa may rupture a thin atrophic membrane c.Pressure by a fluid column ego diving, water sports or forceful syringing d. Fracture of temporal bone Injuries of tympanic membrane may be associated with facial paralysis or subluxation of stapes (vertigo and nystagmus) and sensorineural hearing loss. In such cases, urgent exploration may be required 153. Answer isA . (Surgery): Tubotympanie or Safe type Discharge Perforation Granulations Polyp Cholesteatom a Complication Audiogram Profuse, mucoid, odourless Central Uncommon Pale Absent Rare Mild to Scanty, purulent, foul- smelling Attic or marginal Common Red and fleshy Present moderate conductive deafness Common Conductive or mixed deafness Treatment 1. Tubotympanic type - Aim to control infection and eliminate ear discharge and at a later stage to correct the hearing loss by surgical means (Myringoplasty) 2. Attico-antral type - Surgical is the mainstay of treatment commonly performed operations - atticotomy, modified radical mastoidectomy and rarely mastoidectomy. 154. Answer is D. (Fungal infection): RHINOSPORODIOSIS - It is a fungal granuloma caused by Rhinosporodiumseeberi  The disease mostly affects nose and nasopharynx  **The disease is acquired through contaminated water of ponds and cow-pets  Presents as a leafy, polypoidal mass, pink to purple in colour and attached to nasal septum or lateral wall, sometimes it extends into the nasopharynx and may hang behind the soft palate  The mass is very vascular and bleeds easily on touch it surface is studded with white dots representing the sporangia of fungus  Early stages complains of nasal discharge which is often blood tinged or nasal stuffiness  Sometimes frank epistaxis is the only presenting complaint  *Diagnosis is made by biopsy, not possible to culture Treatment - Complete excision of the mass with diathermy knife and cuaterisation of its base, Dapsone has been tried with some success Rhinoscleroma - caused by Klebsiella -- rhinoscleromatis or French bacillus (Gsram negative bacillus)  Woody feel nose  Diagnosis by - Mikulicz cells and Russell bodies 155. Answer is A. (Surgical): Differences between antrochoanal and ethmoidal polyp Antrochoanal polyp Ethmoidal polyp Age Aetiology Number Laterality Origin Growth Size & shape Common in children Infection Solitary Unilateral Maxillary sinus near the ostium Grows backwards to the choana; may hang down behind the soft palate Tri-lobed with antral, nasal and Common in adults Allergy or multifactorial Multiple Bilateral Ethmoidal sinuses uncinate process, middle turbinate and middle meatus Mostly grow anteriorly and may present at the nares Usually small and grape like Recurrence Treatment choanal parts, choanal part may protrude through the choana fill the nasopharynx obstructing both sides Uncommon, if removed completely Polypectomy, endoscopic removal or Caldwell - Luc operation if recurrent masses Common Endoscopic surgery or ethmoidectomy 156. Answer is A. (Anyrtioteslld): 1. Anterior wall- has a thin plate of bone which seperates the cavity from internal carotid artery. It also has two openings; the lower one from the eustachian tube and the upper one for the canal of tensor tympani muscle 157. Answer is A. (Stapedial paralysis):  Hyperacusis is the sensation of discomfort or pain on exposure to loud noises due to stapedial paralysis or loss of taste (Involvement of chorda tympani)  When the same tone is heard as notes of a different pitch in either ear the condition is known as diplacusis  Bell's palsy - defined as idiopathic, peripheral facial paralysis of acute onset, more common in diabetic (angiopathy) and pregnant women (retension of fluid)  Bell's phenomenon - up and out rolling of the eyeball during forceful closure 158. Answer is A. (Sensorineural deafness not improving with medical treatment): Cochlear Implants - They are electronic devices which converts sound signals into electrical impulses which then directly stimulate the cochlear nerve. Thus they replace the non-functional transducer system of hair cells of the cochlea. 1. Cochlear implants are more useful in post- lingually deaf patients i.e. those who lost their hearing after acquisition of language 2. Congenitally deaf patients have not been benefited as effectively 3. Useful in the totally deaf to give an auditory rhythm signal which can greatly aid lip reading, and are to be thought of as an adjunct to a rehabilitation programme 4. Useful for patients who are unable to benefit from conventional hearing aid. 159. Answer is C. (Surgical removal gives excellent prognosis): Ramsay-Hunt's syndrome I Herpes-Zoster oticus  Herpetic infection of the geniculate ganglion is often associated with facial palsy, accompanying auditory and vestibular symptoms.  Herpetic vesicles on the external ear and the inside the cheek with a LMN facial palsy, can be associated with sensorineural deafness and vertigo.  There may also be anaesthesia of face, giddiness and hearing impairment due to involvement of V th andVIII th nerve.  Treatment is same as Bell's palsy  The prognosis is generally poorer than for a bell's palsy 160. Answer is A. (Loss of corneal reflex): Cerebellopontine angle tumours  Most common tumour is acoustic neuroma (80%) they arise from the Schwann or neurilemmal cells  Commonest nerve of origin is the superior vestibular, followed by the inferior vestibular and rarely the cochlear  Most common presenting symptoms are unilateral deafness of tinnitus or combination of both, 70% have some evidence of ataxia or balance testing  55% have trigeminal nerve impairment, the earliest sign of which is loss of corneal reflex  A lesion at cerehellopontine angle is identified by the presence of vestibular and auditory defects and involvement of other cranial nerves such as 161. Answer is B. (Laryngomalacia): 162. Answer is B. (Mumps): Mumps tends to give a unilateral deafness rather than a bilateral one. Conductive hearing loss Sensorineural hearing loss 1. Negative Rinne test i.e. BC > AC 2. Weber lateralised to poorer ear 3. Normal absolute bone conduction 4. Low frequencies affected more 5. Audiometry shows bone conduction better than air conduction with air-bone gap. Greater the air - bone gap, more is the conductive loss 6. Loss is not more than 60 dB 7. Speech discrimination is good 1. Positive Rinne test i.e. air AC > BC 2. Weber lateralised to better ear 3. Bone conduction reduced on schwaback and absolute bone conduction tests 4. More often involving high frequencies 5. No gap between air and bone conduction curve on audiometry 6. Loss may exceed 60 dB 7. Speech discrimination is poor 8. There is difficulty in hearing in the presence of noise 163. Answer is C. (Subperiosteal abscess): 164. Answer is B. (Hypertrophy of the sebaceous gland): Rhinophyma or potato tumour is a slow growing benign tumour due to hypertrophy of the sebaceous glands of the tip of nose seen in case of long standing acne-rosacea  It presents as a pink, lobulated mass over the nose with superficial vascular dilation mostly affects men past middle age Treatment - Paring down the bulk of tumour with sharp knife or carbon dioxide laser and the area allowed to re-epithelialise. Sometime tumour is completely excised and the raw area skin-grafted. 165. Answer is B. (Tonsillectomy patients): Hypertrophy of lingual tonsils -- Mostly it is a compensatory hypertrophy of lymphoid tissue in response to repeated infections in tonsillectomy patients. Usual complains are discomfort on swallowing feeling of lump in the throat, dry cough and thick voice Treatment - laser surgery 166. Answer is B. (Anterior ethmoid cells (Aggar-nasi): The anterior group sinuses (frontal, anterior, ethmoidal and maxillary) drains into the middle meatus via their ostia (openings) The posterior ethmoidal sinuses drains into the superior meatus and the sphenoid sinus into the sphenoethmoidal recess. At the anterior end of the middle meatus is a ridge - the agger nasi. This represents the nasoturbinal found in many mammals or the "fourth turbinate" 167. Answer is C. (5 -10 mm/min): Mucociliary mechanism - It moves at a speed of 5 - 10 mm per minute and the complete sheet of mucous is cleared into the pharynx every 10 - 20 minutes. • About 600 - 700 ml of nasal secretions are produced in 24 hours 168. Answer is A. (Deafness): ASOM - The principal symptom is pain, sudden and severe, which may waken the child screaming and crying at night. There will usually be a history of a preceding URTI, but this is not invariable. Unless the condition is bilateral or unless there is defective hearing in the other ear deafness may not be noted by the parents. • Hearing tests will show a conductive hearing loss of moderate severity Rinne's test will be negative 169. Answer is C. (Anterior pharyngeal wall): Hypopharynx (Laryngopharynx) is the lowest part of the pharynx and lies behind and partly on the sides of the larynx. Its superior limit is the plane passing from the body of hyoid bone to the posterior pharyngeal wall, while the inferior limit is lower border of cricoid cartilage, where hypopharynx lies opposite the 3'd, 4 th , 5 1 \ 6 th cervical vertebrae  Clinically it is subdivided into three regions 1. Pyriform sinus 2. Post cricoid region 3. Posterior pharyngeal wall 170. Answer is B. (Post-cricoid region): Plummer - Vinson syndrome (Peterson-Brown Kelly Syndrome) consists of dysphagia, hypochromic microcytic anaemia, angular stomatitis glossitis and Koilonychia.  It is a common site for carcinoma in female suffering from plummer-vinson syndrome is Post cricoid region  A curious radiographic finding of unknown cause is a web at the postcricoid region 171. Answer is C. (Astimazolc): Motion-sickness- characterised by nausea, vomiting, pallor, and sweating during sea, air, bus or car travel in certain susceptible individuals.  It can be induced by both real and apparent motion and is thought to arise from the mismatch of information reaching the vestibular nuclei and cerebellum from the visual, labyrinthine and somatosensory systems. It can be controlled by the usual labyrinthine sedative  Prokinetic drugs (Metoclopramide, Domperidone, cisapride, Mosapride) promotes gastrointestinal transit and speed gastric emptying.  In motion sickness - uses sedative antihistaminis, while Astimazole is non sedative anti histaminics 172. Answer is D.(Posterior): The positional vertigos (benign) - Characterized by brief, sudden attacks of vertigo precipitated by head movements. There are no other aural symptoms although the vertigo may be accompanied by a slight felling of nausea. The disease is thought to be due to inorganic deposit on the cupula of the posterior semicircular canals and there is often a preceeding virus type illness. • Similar symptoms can arise after a head injury Central (Malignant) Positional nystagmus - seen with tumours of the posterior cranial fossa or the mid-brain with disseminated sclerosis and with vascular lesions, vetigo is often less than expected 173. Answer is B. (15 - 20 - dB): Rinne's test - In this test air conduction of the ear is compared with its bone conduction. i) Rinne test is cailed positive when AC is longer or louder than BC it is seen in normal persons or those having sensorineural deafness. ii) A negative Rinne (BC> AC) is seen in conductive deafness. A negative Rinne indicates a minimum air-bone gap of 15-20 dB A prediction of air-bone gap can be made if tunning forks of 256, 512 and 1024 Hz are used  A Rinne test equal or negative for 256 Hz but positive for 512 Hz indicates air bone gap of 20 - 30 dB  A Rinne test negative for 256 and 512 Hz but positive for 1024 Hz indicates air-bone gap 000 - 45 dB  A Rinne negative for all the three tuning forks of 256, 512 and 1024 Hz indicates air-bone gap of 45 - 60 dB Remember that a negative Rinne for 256, 512 and 1024 Hz indicates a minimum AB gap of 15, 30,45 dB respectively 174. Answer is B. (1, 3 : 1): 175. Answer is B. (Grants test): Tuning Fork Tests a) Rinne test b) Absolute bone conduction (ABC) c) Schwabach's test d) Weber test e) Bing test f) Gelle's test Ability to understand speech and its relation to speech discrimination score SD score Ability to understand speech 90 -100% Normal 76 - 88% Slight difficulty 66 -74% Moderate difficulty 40 - 58% Poor <40% Very poor 176. Answer is D. (Cerebral hemisphere lesion): Cerebellar hemisphere lesions  Marked ataxia of the trunk  Ipsilateral appendicular deficits (Ataxia, incoordination and hypotonia of the limbs)  Past pointing nystagmus  Dysmetria dysarthria  Gaze-evoked nystagmus is the most common form of jerk nystagmus  Vestibular nystagmus - results from dysfunction of the labyrinth (Minieres disease) vestibular nerve or vestibular nucleus in the brain stem.  Downbeat nystagmus occurs from lesions near the craniocervical junction (chiari malformation, basilar invagination) brainstem or cerebellar stroke lithium or anticonvulsant intoxication, alcoholism and multiple sclerosis  Upbeat nystagmus is associated with damage to the pontinetegmentum from stroke, demyelination or tumours (176-H) 177. Answer is A. (Diplopia): Acute maxillary sinusitis - Most commonly it is viral rhinitis, which spreads to involve the sinus mucosa. This is followed by bacterial invasion. Clinical features 1. Constitutional symptoms - fever, general malaise, and bodyache. 2. Headache 3. Pain - over upper jaw or teeth. 4. Tenderness - Pressure or tapping over the anterior wall of the antrum produce pain 5. Redness and oedema of cheek - commonly seen in children. The lower eyelid may become puffy. 6. Nasal-discharge - Pus may be seen on the upper soft palate on posterior rhinoscopy. 178. Answer is A. (Tissue invasion is seen): Allergic fungal sinusitis - It is an allergic reaction to the causative fungus and presents with sinu- nasal polyposis and mucin. The latter contains eosinophils, Charcot-Leyden crystals and fungal hyphae.  There is no invasion of the sinus mucosa with fungus.  Usually more than one sinus are involved on one or both sides.  ** There may be expansion of the sinus or bone erosion due to the pressure but no fungal invasion.  Treatment is endoscopic surgical clearance of the sinuses with provision of drainage and ventilation.  There is combined with pre and postoperative systemic steroids. Fulminant fungal sinusitis - It is an acute presentation and is mostly seen in immunocompromised or diabetic individuals. Common fungal species are Mucor or Aspergillus A) Mucor causes rhinocerebral disease. Due to invasion of the blood vessels, mucor fungus causes ischaemic necrosis presenting as a black eschar. B) Aspergillus - infection can also cause acute fulminant sinusitis with tissue invasion. Such patients presents with acute sinusitis and develop sepsis and other sinus complications (242-D) 179. Answer is B. (Squamous cell carcinoma): Nasopharyngeal carcinoma - Chinese have higher genetic susceptibility. E. B. virus is closely associated. Air pollution, smoking of tobacco, and opium nitrosamines from dry salted fish, smoke from burning of incense and wood have been incriminated.  Squamous cell carcinoma (85%) is most common  Commonest site of origin in fossa of Rosenmuller in the lateral wall of nasopharynx.  Cervical lymphadenopathy - Most common (60-90%)  Trotter's triad: 1. Conductive deafness (eustachian tube blockage) 2. Ipsilateral temporoparietal neuralgia (Involvement of cranial nerve V) and 3. Palatal paralysis (cranial nerve X)  Irradiation (6000-7000 rads) is the treatment of choice 180. Answer is B. (Glottic carcinoma): Glottic carcinoma - Curative radiotherapy is reserved for early lesions which neither impair cord mobility nor invade cartilage or cervical nodes.  Cancer of the vocal cord without impairment of its mobility gives a 90% cure rate after irradiation and has the advantage of preservation of voice.  Radiotherapy does not give good results in lesions with fixed cords, subglottic extension, cartilage invasion, and nodal metastases. These lesions require surgery. 181. Answer is B. (Modified radical mastoidectomy): Modified radical mastoidectomy - Where as much as the hearing mechanism as possible is preserved. The disease process which is often localized to the attic and antrum is removed and the whole area fully exteriorized into the meatus by removal of the posterior meatal and lateral attic wall. Indication 1. Cholesteatoma confined to the attic and antrum 2. Localized chronic otitis media 182. Answer A As the acoustic neuroma enlarges in cerebellopontine angle first it causes pressure on the fibres of CN V affecting corneal reflex. Motor fibres of CN V are quite resistant. Similarly motor fibres of CN VII are also resistant, though sensory fibres may be affected and cause diminished sensation in posterosuperior wall of external auditory canal (Hitzelberger sign). CN IX and CN X are affected in late stages. 183. Answer is A. (Parapharyngeal abscess): Parapharyngeal abscess (Abscess ofpharyngomaxillary or lateral pharyngeal space) Infections can occure from - Acute and chronic infections of tonsil and adenoid, bursting of peritonsillar abscess, Dental infection usually comes from the lower last molar tooth. Ear, Parotid, retropharyngeal and submaxillary spaces, Trauma. Clinical features Anterior compartment infections produces Posterior compartment involvement TRIAD i) Prolapse of tonsil and tonsillar fossa ii) Trismus (due to spasm of medial pterygoid muscle) iii) External swelling behind the angle of Jaw There is marked odynophagia 1. Bulge of pharynx behind the posterior pillar 2. Paralysis of IX, X, XI, and XII and sympathetic chain 3. Swelling of parotid region. There is minimal trismus or tonsillar prolapse Acute retropharyngeal abscess - Commonly seen in children below 3 years. It is the result of suppuration of retropharyngeal lymph nodes secondary to infection in the adenoids, nasopharynx, posterior nasal sinuses or nasal cavity. In adult may results from penetrating injury. Chronic retropharyngeal abscess results from 1) Caries of cervical spine 2) Tuberculous infection of retropharyngeallymph nodes secondary to tuberculosis of deep cervical nodes. 184. Answer is C (Radiotherapy): Treatment of stridor  Once the diagnosis has been made, treatment of exact cause can be planned.  Oxygen, Tracheostomy, and sometimes steroids may be useful. 185. Answer is A. (Interarytenoid fold): "Pachydermia laryngis" - It is a form of chronic hypertrophic laryngitis affecting posterior part of larynx in the region of interarytenoid and posterior part of the vocal cords  That hypertrophy of the interarytenoid area, formerly known as pachydermia. It is frequently due to acid from oesophageal reflux.  Clinically, patient presents with hoarseness or husky voice, and irritation in the throat.  Sometimes showing ulceration due to constant hammering of vocal processes as in talking forming - called Contact ulcer.  The condition is bilateral and symmetrical. It does not undergo malignant change.  Treatment - is removal of granulation tissue under operating microscope. 186.Answer is D (Brain stem implant): Brain stem implant would be the most appropriate treatment for rehabilitation of a patient who has bilateral profound deafness following surgery for bilateral acoustic schwanoma. Dhingra says "In unilateral acoustic neuroma, ABI is not necessary as hearing is possible from the contralateral side but in bilateral acoustic neuromas as NF2, rehabilitation is required by ABI." Number of questions are increasing in exams regarding rehabilitation of hearing impaired, so I am providing a bit detail of this topic. Important topics related to Hearing Aids CROS: (Contralateral routing of signals)  In this type, microphone is fitted on the side of the deaf ear and the sound picked up is passed to the receiver placed in the better ear. Useful for the person withone ear severely impaired & helps in sound localization comingfrom the side ofdeaf ear Cochlear Implants Electronic devices whichconvert sound signal toelectrical impulses which directly stimulate the fibers of CN VIII. Cochlear implantsreplace the non-functional transducer system of hair cells ofcochlea. Used incochlear type of sensorineural hearing loss where nerve fibers remain intact and functional. • Electrode array consists of multiple electrodes and isimplanted in the scala tympani of the cochlea. It stimulates the fibers of CN VIII. Selection criteria for adults: (post lingually deafened adults are good candidates. Results are not so good in pre-lingually deaf adults unless they had been receiving aural-oral training for communication) A. Age: 18 years or more B. Bilateral severe to profound hearing loss C. Limited or no benefit from hearing aids D. No medical contraindication to undergo surgery Selection criteria for children: (Auditory deprivation in the early developmental period causes degenerative changes in central auditory pathway. Now implants can be used at an age as early age 12 months. Implants can be used in both pre-lingually or post-lingually deafened infantsands children. The criteria are: A. Bilateral profound or severe to profound hearing loss B. Minimal or no benefit from hearing aid C. No medical contraindication D. Willingness and support of family to enroll the child for post-implant training programme Auditory Brainstem Implant (ABI) This implant is designed tostimulate the cochlear nuclear complex in the brainstem directly byplacing the implant in the lateral recess of fourth ventricle. Such implants areneeded when CN VIII has been severed in surgery of vestibular schwannoma. In these cases, cochlear implants are of no use.  In unilateral acoustic neuroma, ABI is not necessary as hearing is possible from trhe contralateral side but in bilateral acoustic neuromas as NF2 rehabilitation is required by ABI. 187.Answer is A (Absent corneal reflex): Dhingra 4th/m True about CP angle tumour is Absent corneal reflex. Acoustic neuroma may present with reduced corneal sensitivity as well as pain, tingling, numbness on face due to involvement of 5th cranial nerve. Acousticneuroma  Acoustic neuroma is a benign, encapsulated, extremely slow growing tumor of the 8th nerve. (Commonest nerve of origin of acoustic neuroma is the Schwann cells of superior vestibular nerve, followed by inferior vestibular and rarely cochlear) Presenting symptom is progressive unilateral Sensorineural hearing loss. This is accompanied by tinnitus. Clinical features 1. Age group of 40-60 years.(M:F=1:1) 2. Cochleo-vestibular symptoms are the earliest symptoms when tumor is still intracanalicular and are caused by pressure on cochlear or vestibular nerve fibers or on internal auditory artery. Progressive unilateral senssorineural hearing loss, often accompanied by tinnitus is the presenting symptom in majority of cases. Cochlear Vestibular Progressive unilateral Sensorineural hearing loss  Tinnitus Difficulty in understanding speech  Imbalance  Unsteadiness  Vertigo 3. Cranial nerve involvement: Vth - Reduced corneal sensitivity and numbness of face Vllth - Hypoasthesia of posterior meatal wall - Loss of taste - Decreased lacrimation IXth&Xth - Dysphagia/hoarseness 4. Brainstem involvement 5. Cerebellar involvement 6. Raised intracranial tension Remember: MRI with gadolinium contrast is the gold standard for diagnosis of acoustic neuroma. Acoustic neuroma can arise from any nerve except Optic & Olfactory because they are myelinated by Oligodendroglia rather than Schwann cells. 188. Answer is A (Single and unilateral): Antro-choanal polyp are usually Single and unilateral. Antrochoanl polyp Ethmoidal Polyp  Age: common in children  Etiology: Infection  Number: Solitary  Laterality: Unilateral  Origin: Maxillary sinus, near the ostium Grows backward to the choana, may hang down behind the soft palate Size & shape: Trilobed with antral, nasal and choanal parts. Choanal part may protrude through the choana & fill the nasopharynx obstructing both sides  Recurrence: uncommon Treatment: polypectomy; endoscopic removal or Caldwell-Luc operation if recurrent Common in adults Allergy or multifactorial  Multiple  Bilateral Ethmoidal sinuses, uncinate process, middle turbinate and middle meatus Mostly grow anteriorly and may present at the noses Usuallysmall and grape like masses  Common Polypectomy; Endoscopic surgery or ethmoidectomy (which may be intranasal, extranasal or transantral) 189. Answer is D (Palatal branch of sphenopalatine): Little's Area or Kisselbach's plexus  Vascular area in the antero-inferior part of nasal septum just above the vestibule Common site of epistaxis Also thesite for origin of the bleeding polypus (hemangioma) of nasal septum Plexus is formed by anastomosis of following vessels:  Anterior ethmoidal  Sphenopalatine  Greater palatine Septal branch of superior labial arteries 190. Answer is B (Mass): Nasopharyngeal carcinoma presents as Mass. Cervical lymphadenopathy (mass) is MC presenting symptom(60-90%) in Nasopharyngeal carcinoma. Nasopharyngeal carcinoma Epidemiology & Geographic distribution: - Nasopharyngeal cancer is most common in China particularly in southern states and Taiwan. - Burning of incense or wood (Polycyclic hydrocarbon), use of preserved salted fish (Nitosamines) along with vitamin C deficient diet (vitamin C blocks nitrosification of amines and and is thus protective) may be other factors operative in China. - People in Southern China, Taiwan and Indonesia are more prone to this cancer Aetiology: Exact etiology is not known. The factors responsible are: - Genetic: Chinese have higher genetic susceptibility to nasopharyngeal cancer. - Viral: Epstein-Barr virus is closely associated with nasopharyngeal cancer. Environmental: Air pollution, smoking of tobacco and opium, nitrosamines from dry salted fish, smoke from burning of incense and wood have all been incriminated. Pathology: Squamous cell carcinoma in various grades of its differentiation or its variants as transitional cell carcinoma and lymphoepithelioma, is the most common. - The commonest site of origin is Fossa of Rosenmuller in the lateral wall of Nasopharynx. Clinical features: Age: It is mostly seen in fifth to seventh decades but may involve younger age groups. Sex: Males are three times more prone than females. Sympatomatology: Cervical lymphadenopathy is MC presenting symptom (60-90%). 1. Nasal: Nasal obstruction, nasal discharge, denasal speech (rhinolaliaclausa) and epistaxis. 2. Otologic: Due to obstruction of Eustachian tube, there is conductive hearing loss, serous or suppurative otitis media. Tinnitus and dizziness may occur. Presence of unilateral serous otitis media in an adult should raise suspicion of nasopharyngeal growth. 3. Ophthalmoneurologic: Involvement ofIXth, Xth and Xlth cranial nerves may occur, constituting jugular foramen syndrome. Nasopharyngeal cancer can cause conductive deafness (Eustachian tube blockage), ipsilateral temporoparietal neuralgia (involvement of CN V) and palatal paralysis (CN X)-collectively called Trotter's triad. 4. Cervical nodal metastases: This may be the only manifestation of nasopharyngeal cancer. A lump of nodes is found between the angle of and the mastoid and some nodes along the spinal accessory in the posterior triangle of neck. Nodal metastases are seen in 75% of the patients, when first seen, about half of them with bilateral nodes. 5. Distant metastases involve bone, lung, liver and other sites. Diagnosis: - Skull X-rays, tomograms or preferably CT scans are done to demonstrate erosion of bone at the base of skull and the extent of tumor. Biopsy is essential to show the exact histology of the malignancy. In absence of nasopharyngeal lesion but with strong suspicion of malignancy, nasopharynx is exposed by transpalatal approach and a strip of mucosa and submucosa from the region of fossa of Rosenmuller should be taken and subjected to histology. Treatment: Irradiation is treatment of choice. 191. Answer is A (Streptococcus): Strep throat: Medline plus, Medical encyclopedia White patch in the throat may be due to Streptococcus. Strep Throat Strep throat is an infection caused by group A Streptococcus bacteria Symptoms • Difficulty swallowing, Fever that begins suddenly •General discomfort, uneasiness or ill feeling • Loss of appetite, Nausea, Rash • Red throat, sometimes with white patches • Sore throat, Tender, swollen lymph nodes in the neck 192. Answer is D (Loss of timbre of voice): Injury to superior laryngeal nerve causes Loss of timbre of voic Paralysis of Superior Laryngeal Nerve Unilateral Bilateral  Paralysis of superior laryngeal nerve causesparalysis of cricothyroid muscle and ipsilateral anaesthesia of larynx above vocal cord. Clinical Features: Voice is weak and pitch cannot be raised Anaesthesia of the larynx on one side may pass unnoticed or causes occasional aspiration  Laryngeal findings includeaskew position of glottis, shortening of cord with loss of tension, flapping of paralyzed cord Both the cricothyroid muscles are paralyzed alongwithanaesthesia of upper larynx Etiology: Surgical or accidental trauma, neuritis (mostly diptheritic), pressure by cervical nodes or involvement in neoplastic process Clinical features:  Presence of bothparalysis and lateral anaesthesia causes inhalation of food and pharyngeal secretions giving rise to cough and choking fits. Voice is weak and husky Treatment:  Cases due toneuritis may recover spontaneously Tracheostomy for repeated aspirations Epiglottopexy (an reversible operation to close the laryngeal inlet to protect the lungs from repeated aspirations) 193. Answer is A (Anterior 1/3 and posterior 2/3 junction): Most common location of vocal nodule Anterior 1/3 and posterior 2/3 junction. Vocal Nodules (Singer's or Screamer's Nodes)  Appearsymmetrically on the free edges of vocal cord  At thejunction of anterior one third, with the posterior two-third, as this is the area of maximum vibration of the cord and subject to maximum trauma Size varies from pin head to half a pea Occur due to vocal trauma when person speaks in unnatural low tones for prolonged periods or at high intensities  They mostly affectteachers, actors, vendors or pop singers Pathology: Trauma to the vocal cord in the form of vocal abuse or misusecausesoedema and hemorrhage in the submucosal space. This undergoes hyalinization and fibrosis, overlying epithelium also undergoes hyperplasia forming a nodule Clinical Features: • Hoarseness, vocal fatigue and pain in the neck on prolonged phonation Treatment Early cases can be treated conservatively (proper use of voice) Surgery is required for large nodules or nodules of long standing in adults, excised with precision under operating microscope •Speech therapy ad re-education in voice productionare essential to prevent their recurrence. 194. Answer is D (Gentamycin): This is the case of otosclerosis and all constitute part of treatment except Gentamycin. Otosclerosis (Otospongiosis)  Disease of bony labyrinth where the normal dense enchondral bone is replaced by foci of irregularly laid spongy bone.  50% patients have positive family history, inherited as autosomal dominant White races are affected more than negroes Female are more common than males Age of onset is around puberty Types of otosclerosis: 1. Stapedial Otosclerosis: Stapedial otosclerosis causingstapes fixation and conductive deafness is the most common variety. Lesion starts just in front of the oval window in an area called fissula ante fenestrum 2. Cochlear Otosclerosis: Involves region of round window 3. Histologic Otosclerosis: remains asymptomatic Clinical Presentation:  Painless & progressive, bilateral (often) conductive hearing loss / deafness is presenting symptom Paracusiswilsii: patient hears better in noisy than quiet environment  Tinnitus, vertigo andmonotonous speech are other features  Tympanic membrane is usually normal and mobile  Schwartz Sign: a reddish hue or Flamingo pink tinge may be seen on the promontory through the tympanic membrane, indicating a focus of increased vascularity. Carhartz notch: A dip in bone conduction may be seen which is different at different frequencies butmaximum at 2000 Hz. Treatment: There is no medical treatment that cures otosclerosis, sodium fluoride has been used to attain maturity of active focus.  Stapedectomy with prosthesis replacement is the treatment of choice  Hearing aid for patients who refuses or unfit for surgery 195. Answer is B (Meningitis): Commonest complication of CSOM is Meningitis. Meningitis  MC intracranial complication of otitis media Can occurin both acute and chronic otitis media In adults, it follows chronic ear disease, which spreads by bone erosion or retrograde thrombophlebitis Clinical features: high grade fever, raised ICT,headache, neck rigidity,projectile vomiting Treatment:  Medical treatment takes precedence over surgery •Meningitis followingacute otitis media may require myringotomy or cortical mastoidectomy. Meningitis following chronic otitis media with cholesteatoma require radical or modified radical mastoidectomy 196.Answer is B (Pyogenic meningitis): Patient presents with high fever, signs of raised ICT and a past history of chronic otitis media likely diagnosis is Pyogenic meningitis. 197.Answer is A (Oval window): The part most commonly involved in otosclerosis is Oval window. 198.Answer is D (Carcinoma larynx): The most common cause of laryngeal stridor in a 60 years old Carcinoma larynx. Glottic cancer is MC site of CA Larynx. Hoarseness is MC and the earliest symptom because of this glottic cancer is detected early. Disease is mostly seen in the age group of 40-70 years. Cancer Larynx  It is 10 times more common in males than females.  Disease is mostly seen in the age group of 40-70 years.  Both tobacco and alcohol are well established risk factor in laryngeal cancer. Cigarette smoke contains benzopyrene and other hydrocarbons which are carcinogenic in man.  About 90-95% of laryngeal malignancies are squamous cell carcinoma with various grades of differentiation. Cordal lesions are often well-differentiated while supraglottic ones are Anaplastic. 1. Supraglottic Cancer: Supraglottic cancer is less frequent than glottic cancer. Nodal metastases occur early, upper and middle jugular nodes are often involved.  Bilateral metastases may be seen in cases of epiglottic cancer. Pain on swallowing is the most frequent initial symptom.  Mass in the neck may be the first sign.  Hoarseness is a late symptom.  Pain may be referred to ear by way of vagus nerve and auricular nerve of Arnold 2.Glottic Cancer: It isMC site of CA Larynx. Mostly originates fromfree edge and upper surface of anterior l/3rd of true vocal cord followed by middle third l/3rd. Spread locally to anterior commissure than to opposite cord(conus elasticus initially acts as barrier for subglottic spread). Fixation of vocal cord indicates spread of disease to thyroaretenoid muscle and is a bad prognostic sign. Asvocal cord is free of lymphatics, nodal metastases is never seen in cordal Cancer, unless the disease spread beyond membranous cord. Hoarseness is MC and the earliest symptom because of this glottic cancer is detected early. Subglottic Cancer: Least common site. Subglottic region extends from glottic area to lower border of cricoid cartilage. Spreads locally around anterior wall to opposite side or down wards to trachea, upward spread to vocal cord is late and hoarseness is not an early symptom. •Lymphatic metastases to prelaryngeal, pretracheal (Delphian node) and lower jugular nodes Earliest and most prominent symptom is stridor, but it appears only in advanced stage. Hoarseness is late feature. 199. Answer is A (Anterior nasal cavity): Internet Thudichum's nasal speculum is used to visualize Anterior nasal cavity. Thudichum's nasal speculum Functions • Used for visualization of external nose - antero-inferior part of nasal septum - anterior part of inferior & middle turbinate 200. Answer is D (Glomus jugular tumour): Pulsatile Tinnitus in ear is due to Glomus jugular tumour. GlomusTumour  MC benign neoplasm of middle ear, origin from glomus bodies Glomus bodies resemble carotid body in structure and are found in dome of jugular bulb or on the promontory along the course of tympanic branch of IXth cranial nerve (Jacobson's nerve) Tumour consists of paraganglionic cells derived from the neural crest Generally seen in the middle age, females are affected five times more Pathology :  Benign, non-encapsulated, extremely vascular neoplasm Rate of growth is very slow and is locally invasive Abundance ofthin walled blood sinusoids with no contractile muscle coat, accounting forprofuse bleeding May invade jugular foramen and the base of the skull, causingIXth to Xllth cranial nerve palsies Clinical Features: When tumour is intra-tympanic: Earliest symptoms are deafness (conductive & slowly progressive) and tinnitus (pulsatile & of swishing character, synchronous with pulse, temporarily stopped by carotid pressure) Otoscopy shows a red reflex through intact tympanic membrane. "Rising sun" appearance is seen when tumor arises from the floor of middle ear. "Pulsatile sign" (Brown's sign) is positive, i.e. when ear canal pressure is raised with Siegle's speculum, tumor pulsates vigorously and then blanches; reverse happens with release of pressure. When tumor presents as a polyp: Profuse bleeding from the ear either spontaneously or an attempts to clean it Examination reveals a red, vascular polyp filling the meatus Phelp's sign: Absence of normal crest of bone between carotid canal and jugular fossa on lateral tomography is virtually diagnostic of glomus jugulare tumor Treatment:  Surgical removal, radiation, embolization or combination Documents Similar To Aiims Adr Plexus EntSkip carouselcarousel previouscarousel nextNeet Psm.aspxEnt Past Years Question According to TopicsPSM - Golden PointsSarp One Linersjipmer_2010_rapid_reviewPhysiology Mock Test Q PaperA Systematic Review of Subjects for PG Medical Entrance ExaminationsSunday 40 Images for August 9 ExamAiims May 2015 Qaent mcq a.pfdorthopedics notes neetpgSurgery One LinersENT QuestionsPgi Chandigarh May 2010 eBookent_MCQENT MCQsAiims May 2015Ent Mcq (All Team) PDFMCQsent mcq 20000ENT MCQs (Girls 1428-29)last minute revision.docENT MCQsMcqs in Ent(2e) Pl DhingraPlatinum Notes - Anatomy ( 2014-15 )Medical Pg NotesAnatomy MnemonicsENT mcqsMCQ EXAMS ENT 3rd edition الهيئة السعوديةMDS Postal Coaching Synopsis Sample MaterialsMore From kishorechandraSkip carouselcarousel previouscarousel nextTestSchedule-testschedule_954504 Pharmacology Pre MedicationMCQ Pharmacology 2321. OrthopaedicsECG-Pg_23-LBBB_Details_(7-16.1-2011) Pathology Mock a.unlockedReproduction Pathology ReviewVaccine StorageACG guidelines for ulcerativecolitis.pdfView Reviews2 Pankaj Jariwalaneuroanatomy mcqsviruses-02-0018915th Aprilviruses-02-00189ANATOMY Multiple Choice Questions 2012Viral InfectionsSkin QuestionsRheumatoid Arthritis Associated Scleritis OPHTHALMOLOGY MCQSMaharastra Paper - 2011_2PGMEEnotes ENTnotes ENT Occasionally Asked TopicsxpressDental Maharastra Key 11Dermatology- Appearances in Various DiseasesSTI Operational Guidelines2004 20HandbookMaharastra Dental Paper - 2014Chapter 29 Capital BudgetingImmunology Aiims Type Exam 1 Fair Game Questions StudyBlueFooter MenuBack To TopAboutAbout ScribdPressOur blogJoin our team!Contact UsJoin todayInvite FriendsGiftsLegalTermsPrivacyCopyrightSupportHelp / FAQAccessibilityPurchase helpAdChoicesPublishersSocial MediaCopyright © 2018 Scribd Inc. .Browse Books.Site Directory.Site Language: English中文EspañolالعربيةPortuguês日本語DeutschFrançaisTurkceРусский языкTiếng việtJęzyk polskiBahasa indonesiaSign up to vote on this titleUsefulNot usefulYou're Reading a Free PreviewDownloadClose DialogAre you sure?This action might not be possible to undo. Are you sure you want to continue?CANCELOK
Copyright © 2024 DOKUMEN.SITE Inc.